Sie sind auf Seite 1von 86

Copyright 2014 Delhi Academy of Medical Sciences, All Rights Reserved.

1/86
Test Information
Test Name SWTS-PSM Total Questions 200

Test Type Examination Difficulty Level Difficult

Total Marks 600 Duration 120minutes

Test Question Language:- ENGLISH

(1). Rashtriya Swastha Bima Yojana is

a. Low cost insurance scheme by general insurance companies

b. Industry associated insurance scheme meant for workers

c. Government health insurance scheme meant for the poor

d. Central Government insurance scheme for employees

Solution. Ans-1: (c) Government health insurance scheme meant for the poor Ref.: RSBY guidelines, Government of India. Sol : RSBY
guidelines,Government of India. With a view to providing health insurance cover to Below Poverty Line (BPL) workers in the unorganised
sector and their families, the Central Government has announced the Rashtriya Swasthya Bima Yojana. Coverage under the scheme
would be provided for
BPL workers and their families [up to a unit of five).
A family would thus comprise the Household Head, spouse, and up to three dependents.
The dependents would include such children and/or parents of the head of the family as are listed as part of the family in the BPL data
base.
Financing of the scheme would be as follows-
(a) Contribution by Government of India: 75% of the estimated annual premium of Rs.750, subject to a maximum of Rs.565 per family per
annum. Additionally, the cost of the smart cards will also be borne by the Central Government @ Rs.60/- per card.
(b) Contribution by the respective State Governments: 25% of the annual premium, as well as any additional premium in cases where the
total premium exceeds Rs.750. (c) The beneficiary would pay Rs.30 per annum as registration/renewal fee. (d) Any administrative and
other related cost of administering the scheme in each State, not otherwise included in the premium cost, shall be borne by the
respective State Governments.

Correct Answer. c

Copyright 2014 Delhi Academy of Medical Sciences, All Rights Reserved. 2/86
(2). A study found that an inverse relation exists between consumption of vegetables containing -carotene and development of cancer. This
finding could also be because of other factors like fibres which are also consumed by people along with the vegetables and which also
prevents the development of cancer. This is an example of

a. Differential misclassification

b. Confounding

c. Regression of mean

d. Multifactorial association

Solution. Ans-2: (b) Confounding Ref.: Epidemiology, Leon Gordis, 2nd Edition, 204-217 Sol :
Bias: Any systematic error in the design, conduct or analysis of a study that results in a mistaken estimate of an exposures effect on the
risk of disease.
1.Selection Bias- If the way, in which cases and controls , or exposed and non exposed individuals are selected in such that an apparent
association is observed- even if in reality exposure and disease are not associated- the apparent association is the result of selection bias.
The nature of the selection potentials affects the generalizability or external validity of the study. 2.Information Bias: This bias can occur
when the means for obtaining information about the subjects in the study are inadequate so that as a result some of the information
gathered regarding exposures and/or disease outcome is incorrect. Given inaccuracies in the methods of data acquisition, we may at
times misclassify subjects, and thereby introduce misclassification bias. Misclassification bias can occur in two ways:
Differential
Non differential.
Misclassification that occurs in the same proportion in each group is called nondifferential or random misclassification. For example,
suppose the outcome in a study of exercisers and non-exercisers is MI. If 20% of exercisers were erroneously classified as having a MI,
and 20% of the non-exercisers were erroneously classified as having a MI, nondifferential misclassification of the outcome occurred. The
usual effect of non differential misclassification is that the RR or OR tends to be diluted and it is shifted towards one. In other words we
are less likely to detect an association, even if it really exists.
Misclassification that occurs in different proportions in each group is called differential or non-random misclassification. Using the same
example in the paragraph above, if 20% of exercisers were erroneously classified as having a MI, and 5% of the non-exercisers were
erroneously classified as having a MI, differential misclassification of the outcome occurred. So, a differential misclassification bias can
lead either to an apparent association even if one does not really exist or to an apparent lack of association when one does exist.
3.Confounding: Confounding occurs when two conditions are met- (1) There must be an association between a variable (third factor) and
the exposure status. The confounding variable is associated with the exposure but it is not the result of the exposure. In the diagram
below, smoking is associated with drinking alcohol but it is not the result of drinking alcohol. (2) The variable must also be an
independent risk factor for the outcome. An additional caveat is that the variable cannot be an intermediate between the exposure and
the disease, e.g. cigarette smoking to pre-cancerous lung cells to cancerous lung cells. In this case, the presence of pre-cancerous lung
cells is an intermediate, not a confounder.

Correct Answer. b

Copyright 2014 Delhi Academy of Medical Sciences, All Rights Reserved. 3/86
(3). According to Workmen's Compensation Act 1923, which of the following is included as occupational disease?

Copyright 2014 Delhi Academy of Medical Sciences, All Rights Reserved. 4/86
a. Typhoid

b. Tetanus

Copyright 2014 Delhi Academy of Medical Sciences, All Rights Reserved. 5/86
c. Anthrax

d. Dengue

Solution. Ans-3: (c) Anthrax Ref.: http://lawcommissionofindia.nic.in/51-100/Report62. Sol : List of Occupational Diseases as per Employee
Compensation Act 1923 (earlier Workmen Compensation Act, 1923)

Correct Answer. c

Copyright 2014 Delhi Academy of Medical Sciences, All Rights Reserved. 6/86
(4). If child survival rate is 77.5, calculate. Under 5 mortality rate.

a. 275

b. 225

c. 775

d. 7.75

Solution. Ans4: (b) 225 Ref: Park -423.

Correct Answer. b

(5). Study the given table: Calculate Odd Ratio CASE CONTROL STUDY OF SMOKING AND LUNG CANCER

a. 8.1

b. 4.05

c. 2.025

d. 1

Solution. Ans 5: (b) 4.05 Reference P-64 SOL Odds Ratio


It is a measure of the strength of the association between risk factor and outcome
Odds ratio is closely related to relative risk

Correct Answer. b

Copyright 2014 Delhi Academy of Medical Sciences, All Rights Reserved. 7/86
(6). Countries implementing DOTS strategy in over 90 percent of the total population (routine implementation):

a. Category1

b. Category 2

c. Category 3

d. Category4

Solution. Ans 6: (d) Category 4 Ref P -146.


SOL

Correct Answer. d

(7). Leprosy is not targeted for global eradication because

a. No effective vaccine

b. Highly infectious but low pathogenicity

c. Long incubation Period

d. Only Humans are reservoir

Solution. Ans-7: (b) Highly infectious but low pathogenicity Ref.: Parks - 288 Sol :
The Government of India had declared its intention to eradicate leprosy by the year 2000 A.D.
Except for small pox, no other disease has been eradicated to date.
The eradication of small pox has been singularly instructive.
It has taught us that there are certain epidemiological criteria which sould be met before a disease qualifies for eradication, i.e.
o There should be no known animal reservoir
o There should be no long-term carriers of the organism
o Life-long immunity is attained after recovery from the disease
o The detection of cases should be comparatively simple
o Persons with subclinical infection should not transmit the disease
o Vaccine should be highly effective, easily administered, heat-stable and should confer long-term protection and
o International co-operation
Leprosy falls far short of these requirements and therefore, is not a suitable candidate for eradication.
The real hurdles against leprosy which small pox did not pose are
o The long and variable incubation period
o The disputed modes of transmission
o The presence of subclinical cases and our inability to detect them in the community.
o The complicated spectrum of disease manifestations
o The failure of cell mediated immunity in lepromatous cases
o Bacterial resistance and persistence in the human body
o Absence of a vaccine
o Social and cultural taboos leading to concealment of the disease
o Discovery of extra-human reservoirs

Correct Answer. b

Copyright 2014 Delhi Academy of Medical Sciences, All Rights Reserved. 8/86
(8). The re-evaluation of a case of acute flaccid paralysis for residual paralysis for the purpose of AFP surveillance is done after how many
days?

a. 60 days

b. 90 days

c. 6 months

d. 1 year

Solution. Ans-8: (a) 60 days Ref.: Parks Textbook of Preventive and Social Medicine; 21/e, pg183 Sol : AFP surveillance
Acute onset flaccid paralysis in child < 15 yrs age.
Notification DIO / SMO.
2 specimens at least 24 hrs apart within 14 days of onset adequate volume 8-10 gm (thumb size).
Active search for AFP cases.
F/U examination after 60 days for residual paralysis.
All activities within 70 days.
AFP surveillance is evaluated by 2 key indicators-
The sensitivity of reporting (target being non-polioAFP rate of atleast 1 case per 100,000 children aged less than 15 years).
The completeness of specimen collection(target being 2 adequate stool specimens from atleast 80% of all cases).

Correct Answer. a

(9). What is the recommended ophthalmologist:population ratio in India according to vision 2020?

a. 1:5,000,000

b. 1:1,00,000

c. 1:50,000

d. 1:10,000

Solution. Ans-9: (c) 1:50,000 Ref.: http://www.aios.org/cme/cmeseries9.pdf Sol :

Correct Answer. c

(10). Minimum microbiological criteria for water quality includes

a. All samples should be free of coliforms

b. Not more than 5% samples should have any coliform

c. Any 100 ml should not have more than 3 E. coli

d. Coliforms should not be detectable in 3 consecutive samples of 100 ml of water

Solution. Ans-10: (a) All samples should be free of coliforms Ref: Parks- 631 Sol :
In all water intended for drinking, E. coli or thermotoleranat coliform bacteria must not be detectable in any 100 ml sample.

Correct Answer. a

Copyright 2014 Delhi Academy of Medical Sciences, All Rights Reserved. 9/86
(11). Additional daily energy requirement for a lactating woman with a two month old child will be

a. 450 calories

b. 550 calories

c. 650 calories

d. 750 calories

Solution. Ans-11: (b) 550 calories Ref: Parks- 548 Sol :


The energy requirements of women are increased by
Pregnancy (+ 300 kCal daily throughout pregnancy ) and
Lactation (+ 550 kCal adily during the first 6 months, and + 400 kCal daily during the next 6 months.

Correct Answer. b

(12). According to vision 2020 the recommendation for secondary care services including cataract surgery is

a. 1 service centre for 5000 population

b. 1 service centre for 50,000 population

c. 1 service centre for 5,00,000 population

d. 1 service centre for 5,000,000 population

Solution. Ans-12: (c) 1 service centre for 5,00,000 population


Ref.: All India Ophthalmological society website- http://www.aios.org/ http://www.cehjournal.org/indian/journal/18/jceh_18_54_s061.html
Sol :
1 service centre for 5,00,000 population.
1 vision center for primary health care is recommended for a population of 50,000 and for every 10 vision center there is one service
center providing secondary care services.

Correct Answer. c

Copyright 2014 Delhi Academy of Medical Sciences, All Rights Reserved. 10/86
(13). Bias can be eliminated by all except

a. Matching

b. Blinding

c. Randomization

d. Multivariate analysis

Solution. Ans-13: (d) Multivariate analysis Ref.: Parks Textbook of Preventive and Social Medicine; 20/e, pg 69-70, 78, Clinical
Epidemiology. The Essentials. Robert H Fletcher. Fourth Edition. Table 7.3, Page 119 Sol :
Bias is any systematic error in the determination of the association between the exposure and the disease.
There are various methods for controlling bias, which includes Randomisation, Restriction, Matching, Stratification, Simple adjustment,
Mutlivariate analysis and Blinding.
So, in this question, all the four options hold good for explaining the concept of Elimination of Bias.
But as one of the option is to be chosen, one can select Multivariate analysis as Matching, Blinding and randomization are carried out
during the design of study while Multivariate analysis is done during the analysis.

Correct Answer. d

(14). True regarding the role played by a social worker is

a. Health professional responsible for functional evaluation and activities of the patient

b. Health professional responsible for instituting therapeutic exercises and mobility training of the patient

c. Health professional involved in patients personal role performance, interpersonal skills and family functions

d. He establishes and sustains the people with disability for employment

Solution. Ans-14: (c) Health professional involved in patients personal role performance, interpersonal skills and family functions Ref.:
Read the text below Sol :
In Given choices option (3) best defines the role of Social woeker.
Option (2) Health professional responsible for instituting therapeutic exercises and mobility training of the patient- defines role of
physiotherapist
Option (4) He establishes and sustains the people with disability for employment- defines role of occupational therapist.

Correct Answer. c

Copyright 2014 Delhi Academy of Medical Sciences, All Rights Reserved. 11/86
(15) Filaria elimination is possible in South-east Asian region due to all of
. the following except

a. Humans are the only reservoir

b. Parasite does not multiply in the intermediate host mosquito

c. Larvae multiply in human reservoir only

d. Infectious larvae gets deposited on skin but most of them die


before penetrating

Solution. Ans-15: (a) Humans are the only reservoir


Ref.: http://www.searo.who.int/en/Section10/Section2096_10600.htm
Sol :

Correct Answer. a

(16). Under RNTCP , a new case is one who has never had treatment for TB or has taken anti TB drugs for less than:

a. 2 weeks

b. 4 weeks

c. 8 weeks

d. 6 months

Solution. Ans 16: (2) 4 weeks Reference P -149. Sol: IMPORTANT DEFINATION REGARDING TUBERCULOSIS Smear positive
tuberculosis: At least two initial sputum smears positive for AFB or one AFB positive smear and positive culture. Smear negative
tuberculosis: At least three negative smears, but tuberculosis suggestive symptoms and X-ray abnormalities or positive culture.
Adherence: Person take appropriate drug regimen for required time also known as compliance. New Case: A patient with sputum positive
pulmonary tuberculosis who has never had treatment for tuberculosis or has taken anti-tuberculosis drugs for less than 4 weeks.
Relapse: A patient who returns smear positive having previously been treated for tuberculosis and declared cured after the completion of
this treatment. Return after default: A patient who returns sputum smear positive, after having left treatment for at least two months.

Correct Answer. b

Copyright 2014 Delhi Academy of Medical Sciences, All Rights Reserved. 12/86
(17) True statement regarding Culicini Culex:.
.

a. Siphon tube in larvae

b. Wings in adults are spotted

c. Eggs are boat shaped

d. Pupae have broad and short

Solution. Ans17: (1) Siphon in larvae Ref: Sol:Read the text below

Correct Answer. a

(18). For the field diagnosis of trachoma ,WHO recommends that follicular and intense trachoma inflammation should be assessed in:

a. Women aged 15-45 yrs

b. Women aged >45 yrs

c. Children aged 0- 5 yrs

d. Children aged 0-10 yrs

Solution. Ans18: (c) Children aged 0-10 yrs Ref Read the text below Sol.
A prevalence of more than 5 percent severe and moderate trachoma in children
under 10 years is an indication for mass or blanket treatment.
The treatment consists of the application twice daily of tetracycline 1 percent ointment to all children, for 5 consecutive days each
month or once daily for 10 days each month for 6 consecutive months, or for 60 consecutive days.
An alternative antibiotic is erythromycin

Correct Answer. c

Copyright 2014 Delhi Academy of Medical Sciences, All Rights Reserved. 13/86
(19). Extended Sickness benefit under ESI Act, is available for all, except.

a. Congestive Heart Failure

b. Chronic Obstructive Lung Disease

c. Ankylosing spondylitis

d. Cardiomyopathies

Solution. Ans19: (c) Ankylosing spondylitis Ref Read the text below

Correct Answer. c

(20). Regarding Dengue Haemorrhagic Fever, hypotension with the presence of cold clammy skin and restlessness is included in:

a. Grade I

b. Grade II

c. Grade III

d. Grade IV

Solution. Ans 20:(c) Grade III Reference - Read the text below Sol. Grading of severity of DHF The severity of DHF has been classified
into four grades
Pathophysiological hallmark- shock and bleeding.
The presence of thrombocytopenia with concurrent haemoconcentration differentiates grade I and Grade II DHF from DF and other
disease
Grade I: Fever accompanied by non-specific constitutional symptoms. The only haemorrhagic manifestation is a positive tourniquet test.
Grade II: Patient with spontaneous bleeding usually in the form of skin and/or other haemorrhages in addition to the manifestations in
Grade-I. Grade III: Circulatory failure manifested by repaid and weak pulse, narrowing of pulse pressure (20 mm Hg. or less) or
hypotension with the presence of cold clammy skin and restlessness. Grade IV: Profound shock with undetectable blood pressure and
pulse

Correct Answer. c

Copyright 2014 Delhi Academy of Medical Sciences, All Rights Reserved. 14/86
(21). Bayes theorem is:

a. As the prevalence of a disease increases, the positive predictive valve (PPV) of the test increases and its negative predictive valve
decreases

b. As the prevalence of a disease increases, the negative predictive valve (PPV) of the test increases and its negative predictive valve
decreases.

c. As the prevalence of a disease decreases, the positive predictive valve (PPV) of the test increases and its negative predictive valve
decreases.

d. As the prevalence of a disease increases, the positive predictive valve (PPV) of the test increases and its negative predictive valve
increases.

Solution. Ans 21 : (a) As the prevalence of a disease increases, the positive predictive valve (PPV) of the test increases and its negative
predictive valve decreases. Ref:Read the text below Sol. Bayes theorem is:As the prevalence of a disease increases, the positive
predictive valve (PPV) of the test increases and its negative predictive valve decreases.

Correct Answer. a

(22). The basic health workers were to be utilised for purposes other than family planning was recommended by ?

a. Chadah Committee

b. Jungalwalla Committee

c. Mukherji Committee

d. Mudaliar Committee

Solution. Ans 22: (c) Mukherji Committee Ref:Read the text below Sol. Mukherjee Committee
The committee recommended separate staff for the family planning programme.
The family planning assistants were to undertake family planning duties only.
The basic health workers were to be utilised for purposes other than family planning.
Recommended to delink the malaria activities from family planning.

Correct Answer. c

Copyright 2014 Delhi Academy of Medical Sciences, All Rights Reserved. 15/86
(23). Five modes of intervention have been described which form a continuum corresponding to the natural history of any disease.These levels
are related to agent,host and environment.Example of activities outside the health sector having a bearing on health promotion is: -

a. Family life education.

b. Inculcation of healthy habits

c. Increasing overall socioeconomic development

d. Chlorination of water

Solution. Ans 23: (c) Increasing overall socioeconomic development Reference Gupta/mahajan-9 Sol:

Correct Answer. c

(24). Fluoride level requirement is :

a. 0.32-0.44 mg/dl

b. 0.2-0.9 mg/dl

c. 0.5-0.8 mg/dl

d. 0.23-0.55 mg/dl

Solution. Ans-24: (c) 0.5-0.8 mg/dl Ref: Read the text below Sol : Optimum Fluoride Requirement
The recommended level of fluorides in drinking water in this country is accepted as 0.5 to 0.8 mg per litre.
In temperate countries where the water intake is low, the optimum level of fluorides in drinking water is accepted as 1 to 2 mg per litre.

Correct Answer. c

Copyright 2014 Delhi Academy of Medical Sciences, All Rights Reserved. 16/86
(25). As compared to a routine case control study, nested case control study avoids problems (in study design) related to

a. Temporal association

b. Confounding bias

c. Need for long follow up

d. Randomization

Solution. Ans-25: (a) Temporal association Ref: Read the text below Sol :
Nested case control study is a type of cohort study. In a nested case-control study, cases of a disease that occur in a defined cohort are
identified and, for each, a specified number of matched controls is selected from among those in the cohort who have not developed the
disease by the time of disease occurrence in the case.
Nested case-control design potentially offers impressive reductions in costs and efforts of data collection and analysis compared with the
full cohort approach, with relatively minor loss in statistical efficiency.
This study design is particularly advantageous for studies of biologic precursors of disease. Compared with case control study, , nested
case-control studies can reduce recall bias ' and temporal ambiguity, and compared with cohort studies, can reduce cost and save time.
The drawback of nested case-control studies is non-diseased persons from whom the controls are selected may not be fully
representative of the original cohort, due to death or failure to follow-up cases.

Correct Answer. a

(26). You have diagnosed a patient clinically as having SLE and ordered 6 tests. Out of which 4 tests have come positive and 2 are negative. To
determine the probability of SLE at this point, you need to know :

a. Prior probability of SLE; sensitivity and specificity of each test

b. Incidence of SLE and predictive value of each test

c. Incidence and prevalence of SLE

d. Relative risk of SLE in this patient

Solution. Ans-26: (a) Prior probability of SLE; sensitivity and specificity of each test Ref: Read the text below Sol :
Prior probability of SLE; sensitivity and specificity of each test One has to answer the following question first. What is the likelihood
that this patient has SLE when the test result is positive? and What is the likelihood that this patient does not have the SLE when the
test result is negative? The answers to these questions are known as the positive predictive values (PPV) and negative predictive values,
(NPV) respectively. The PPV and NPV can also be calculated from Bayes' theorem. Bayes' theorem allows us to compute the PPV and
NPV from estimates of the test's sensitivity and specificity, and the probability of the disease before the test is applied. The latter is
referred to as the pretest probability and is based on the patient's previous medical history, previous and recent exposures, current signs
and symptoms, and results of other screening and diagnostic tests performed. When this information is unknown or when calculating the
PPV or NPV for a population, the prevalence of the disease in the population is used as the pretest probability. The PPV and NPV, then,
are called posttest probabilities (also, revised or posterior probabilities), and represent the probability of the disease after the test result
is known.

Correct Answer. a

Copyright 2014 Delhi Academy of Medical Sciences, All Rights Reserved. 17/86
(27). The concept of design effect is applied toa.

a. Stratified Sampling

b. Systematic Sampling

c. Cluster Sampling

d. Simple random Sampling

Solution. Ans-27: (c) Cluster Sampling Ref: Read the text below Sol :
Concept of Design Effect is applied to Cluster Sampling. Cluster sampling is a commonly used technique when the population is spread
out, and the researcher cannot sample from everywhere. However, respondents in the same cluster are likely to be somewhat similar to
one another. As a result, in a clustered sample Selecting an additional member from the same cluster adds less new information than
would a completely independent selection. Thus, for example, in single stage cluster samples, the sample is not as varied as it would be
in a random sample, so that the effective sample size is reduced. The loss of effectiveness by the use of cluster sampling, instead of
simple random sampling, is the design effect. The design effect is basically the ratio of the actual variance, under the sampling method
actually used, to the variance computed under the assumption of simple random sampling. For an example, The interpretation of a value
of (the design effect) of, say, 3.0, is that the sample variance is 3 times bigger than it would be if the survey were based on the same
sample size but selected randomly. An alternative interpretation is that only one-third as many sample cases would be needed to measure
the given statistic if a simple random sample were used instead of the cluster sample with its (design effect) of 3.0. The main components
of the design effect are the intra class correlation, and the cluster sample sizes. Thus, the design effect is calculated as follows: DEFF = 1
+ (n 1), where DEFF is the design effect, is the intraclass correlation for the statistic in question, and , n is the average size of
the cluster. It can be seen that the design effect increases as the cluster sizes increase, and as the intra class correlation increases.

Correct Answer. c

(28). Children between the ages of 9 and 13 can work maximum how much work/day according to the Factories Act?

a. 6 hours

b. 8 hours

c. 7 hours

d. 4 hours

Solution. Ans-28: (a) 6 hours Ref: Read the text below Sol :
The Factories Act 1802 was an Act of the Parliament of the United Kingdom which regulated factory conditions, especially in regard to
child workers in cotton and woollen mills. It was the culmination of a movement originating in the 18th century, where reformers had
tried to push several acts through Parliament to improve the health of the workers and apprentices. The act had the following provisions:
Factory owners must obey the law.
All factory rooms must be well ventilated and lime-washed twice a year.
Children must be supplied with two complete outfits of clothing.
Children between the ages of 9 and 13 can work maximum 8 hours.
Adolescents between 14 and 18 years old can work maximum 12 hours.
Children under 9 years old are not allowed to work but they must be enrolled in the elementary schools that factory owners are required
to establish.
The work hours of children must begin after 6 a.m., end before 9 p.m., and not exceed 12 hours a day.
Children must be instructed in reading, writing and arithmetic for the first four years of work.
Male and female children must be housed in different sleeping quarters.
Children may not sleep more than two per bed.
On Sundays children are to have an hour's instruction in Christianity.
Factory owners are also required to tend to any infectious diseases.

Correct Answer. a

Copyright 2014 Delhi Academy of Medical Sciences, All Rights Reserved. 18/86
(29). Operational efficacy of malaria is measured by :

a. API

b. Infant Parasite Rate

c. A and B

d. ABER

Solution. Ans-29: (d) ABER Ref: Read the text below Sol :
ABER - Annual Blood Examination Rate. Calculated as (number of slides examined/population) x 100. WHO recommendation for
malarious areas is that the number of slides examined per month should equal at least 1% of the population.
API - Annual Parasite Incidence. API = (confirmed cases during 1 year/population under surveillance) x 1000.

Correct Answer. d

(30). Child survival rate includes which one of the following?

a. Children below 5 years

b. Children below 4 years

c. Children up to 1 year

d. All of the above

Solution. Ans-30: (a) Children below 5 years Ref: Read the text below Sol :

Correct Answer. a

(31). Choose the incorrect statement regarding Q fever :

a. There is an arthropod involved in its transmission to man

b. Infection may occur through abrasions or conjunctivae

c. An inactivated Coxiella vaccine has also been prepared

d. Tetracyclines orally for a period of at least 5 days after remission of fever is recommended

Solution. Ans 31: (a) There is an arthropod involved in its transmission to man Reference Read the text below Sol:
Q FEVER Agent factors Agent
Coxiella burnetii
Vector Ticks
Source of infection
(a) Faeces and urine ( Cattle, Sheep , Goats , Ticks )
(b) Placenta of infected animals
Mode of transmission There is no arthropod involved in its transmission to man Modes:
(a) Inhalation of infected dust from soil
(b) Through abrasions,conjunctivae or ingestion of infected food
(c) Respiratory route
Incubation period
2 to 3 weeks
Clinical features
(a) Acute onset with fever,chills,general malaise and headache
(b) May cause pneumonia,hepatitis,encephalitis and rarely endocarditis
Control measures Treatment: Tetracyclines orally for a period of at least 5 days after remission of fever. PREVENTNE MEASURES:
An inactivated Coxiella vaccine has also been prepared to protect occupationally exposed workers
Pasteurization or boiling of milk to inactivate the causative agent.
Providing sanitary cattle sheds ,adequate disinfection and disposal of products.

Correct Answer. a

Copyright 2014 Delhi Academy of Medical Sciences, All Rights Reserved. 19/86
(32). All the features characterizes chickenpox infection except : -

a. Virus can be isolated from vesicular fluid during the first 3 days of illness

b. The fever does not run high

c. Scrabs begins to form 10-14 days after the rash appears

d. Evolution of rash is very rapid

Solution. Ans 32: (c) Scrabs begins to form 10-14 days after the rash appears Reference P-125 Sol: CHICKENPOX:
Virus can be isolated from vesicular fluid during the first 3 days of illness.
Scabs however are not infective.
The fever does not run high.

Correct Answer. c

Copyright 2014 Delhi Academy of Medical Sciences, All Rights Reserved. 20/86
(33). The mid -day meal programme (MDMP),also known as School Lunch Programme has been in operation since 1961. Meal should supply at
least :

a. One-third of the total energy requirement, and one-third of the protein need

b. One-third of the total energy requirement, and half of the protein need

c. Half of the total energy requirement, and half of the protein need

d. Half of the total energy requirement, and one-third of the protein need

Solution. Ans 33: (2) One-third of the total energy requirement, and half of the protein need Ref:Read the text below Sol:
Meal should supply at least one-third of the total energy requirement, and half of the protein need

Correct Answer. b

(34). Change in incidence of an infective disease is best judged in relation to incidence in the past as reflected in monthly incidence records;
Considerable incidence will be considered if : -

a. 1-3 new cases/100,000/week occur

b. 3-5 new cases/100,000/week occur

c. 5-10 new cases/100,000/week occur

d. 10-15 new cases/100,000/week occur

Solution. Ans34: (b) 3-5 new cases/100,000/week occur Reference Gupta/mahajan-19 Sol: The following terms are used to express
various grades of incidence and prevalence of communicable diseases in a community. 1. Mild incidence: When less than 3 new cases per
week are reported in 100,000 population. 2. Considerable incidence: When 3 to 5 new cases/ 100,000/week occur. 3. Heavy incidence:
When 5 to 10 new cases/l00,000/ week are reported.

Correct Answer. b

(35). Main disadvantage of Pyrolytic incineration is : -

a. Incomplete destruction of cytotoxics.

b. Significant emissions of atmospheric pollutants

c. Need for periodic removal of slag and soot

d. Requires highly qualified technicians.

Solution. Ans35 : (1) Incomplete destruction of cytotoxics. Reference P19-647 Sol:

Correct Answer. a

Copyright 2014 Delhi Academy of Medical Sciences, All Rights Reserved. 21/86
(36). The government of India have initiated several large scale supplementary feeding programmes,an programmes aimed at overcoming
specific deficiency diseases through various Ministries to combat malnutrition,programme such as Midday meal programme was initiated
by:

a. Ministry of Health

b. Ministry of Family welfare

c. Ministry of Social welfare

d. Ministry of Education

Solution. Ans36: (d) Ministry of Education Reference P-526 Sol:

Correct Answer. d

(37). If perinatal mortality is higher in hospital births than in the home births it is an example of :-

a. Spurious association

b. Indirect association

c. One-to-one association

d. Multifactorial association

Solution. Ans37: (a) Spurious association Reference P19-82 Sol: SPURIOUS ASSOCIATION
Sometimes an observed association between a disease and suspected factor may not be real.
E.g the perinatal mortality is higher in "hospital births than in the home births.
It might be concluded that homes are a safer place for delivery of births than hospitals. Such a conclusion is spurious or artifactual,
because in general, hospitals attract women at high risk for delivery because of their special equipment and expertise, whereas this is
not the case with home deliveries

Correct Answer. a

Copyright 2014 Delhi Academy of Medical Sciences, All Rights Reserved. 22/86
(38). Regarding Bio Medical wastes Microbiology and Biotechnology waste are included in:

a. Category no.2

b. Category no.3

c. Category no.4

d. Category no.5

Solution. Ans38: (b) Category no.3 Reference Read the text below Sol:

Correct Answer. b

(39). Consider the following data Regarding HDI ,minimum and maximum values of life expectancy at birth is 25 yrs and 85 yrs respectively.If
life expectancy at birth for India is 62.6 yrs,then calculate life expectancy index: -

a. 0.535

b. 0.626

c. 0.55

d. 0.47

Solution. Ans39: (b) 0.626 Reference P19-17 Sol: Human Development Index Human development index (HDI) is defined as "a
composite index combining indicators representing three dimensions - longevity (life expectancy at birth); knowledge (adult literacy rate
and mean years of schooling); and income (real GDP per capita in purchasing power parity in US dollars)"

If; Minimum and maximum values of life expectancy at birth is 25 yrs and 85 yrs respectively Life expectancy at birth for India is 62.6
Then

Correct Answer. b

Copyright 2014 Delhi Academy of Medical Sciences, All Rights Reserved. 23/86
(40). The National institute of nutrition is of the view that the minimum number of feeding days in the year should be:

a. 275days

b. 250 days

c. 310 days

d. 325 days

Solution. Ans 40: (b) 250 days Ref:Read the text below Sol: The National Institute of Nutrition is of the view that the minimum number
of feeding days in a year should be 250 to have the desired impact on the children

Correct Answer. b

(41). All are true about reference Indian female except

a. 20-39 years of age

b. 55 kg weight

c. 2 hours active, recreational, household work

d. 8 hours of bed sleep

Solution. Ans 41: (b) 55 kg weight Ref: Read the text below Sol: Criteria for Normal Indian Reference Female
Weighs 50 kg.
Spends 8 hours in bed, 4-6 hours sitting and moving around and 2 hours in 20-39 years.
Free from disease and physically fit for active work on each working day, 8 hrs household work or light industry or moderate active work
Criteria for Normal Indian Reference Male
Weighs 60 kg.
Spends 8 hours in bed, 4-6 hours sitting and moving around and 2 hours in walking and in active recreation or household duties.
20-39 years.
Free from disease and physically fit for active work on each working day, employed for 8 hrs that involves moderate activity. A reference
male term is given to decide one who takes 1 unit, which is equivalent to 2425 calories so 2200 cal is not reference
Other important tables

Correct Answer. b

Copyright 2014 Delhi Academy of Medical Sciences, All Rights Reserved. 24/86
(42). Kaplan Meire test is used for

a. Survival

b. Incidence

c. Prevalence

d. Frequency

Solution. Ans 42: (a) Survival Ref: pdq biostatistics pg 116-120) Sol: Kaplan Meire curves are also known as survival curves
There are methodologic problems that require the use of a different form of analysis, usually called life table analysis or survival curve
analysis.
Patients are usually enrolled into the study over a protracted period of time.there are always less no of patient at one point of time who
can start trial at same time with long studies there are drop out and we have no way what happened to them. Another thing is patient die
before the completion of study.
what do we do with the withdrawals? Obviously, we can't determine length of survival from their data, because of the different survival
and loss of substantial patients from study (due to drop out and death) so we have to discard lot of our data, withdrawals did provide
useful information before they withdrew.
So we have two method for calculation of survival:
Actuarial method for survival analysis, in which the status of the subjects is looked at after a fixed period of time (each month, or every 6
months, or every year, depending on the length of follow-up and how frequently the end points occur). slght modification of the
technique, called the Kaplan-Meier (K-M) approach.
The major difference between it and the actuarial method is that calculating the probability of survival only at the fixed intervals, the K-
M analysis calculates it only when an end point occurs. So it's possible that there may be 6 months between the first and second analyses
but only 2 days between the second and third.
The K - M method is usually more efficient than the actuarial method when there are fewer than 50 subjects, but the differences
between the results are quite minor for the most part.

Correct Answer. a

(43). If 10th percentile of weight of 200 boys of age 5 yrs is 16 kg then it means that :

a. 18 boys have weight more than 16 kg.

b. 18 boys have weight less than 16 kg.

c. 18 boys have weight less than 16 kg.

d. 180 boys have weight more than 16 kg

Solution. Ans 43: (d) 180 boys have weight more than 16 kg. Reference Biostatistics by A.indrayan --Just as median divides the
subjects in two equal groups, each with nearly n/2 subjects, the percentiles divides the subjects in 100 equal parts, each part containing
n/100 subjects.
The parts are identified by 99 cutpoints of the measurements under consideration.
e.g. 10th percentile of weight is that value below which are 10 percent children.
For n=200, 10th percentile of children mean 20 children therefore 20 children are below the value of 16 kgs or 180 children will be
having weight more than 16 kgs.

Correct Answer. d

Copyright 2014 Delhi Academy of Medical Sciences, All Rights Reserved. 25/86
(44). Which of the following committees promoted the concept of urban primary health care?

a. Bhore

b. Shrivastav

c. Kartarsingh

d. Krishnan

Solution. Ans-44: (d) Krishnan Ref.: Read the text below Sol :
More than one quarter of the population in the country now lives in urban areas. In metropolitan and large cities about 40-50% of the
urban dwellers are estimated to be living in slum areas where the health status of the people is as bad as, if not worse than, in rural
areas.
But infrastructure for primary health care in urban areas hardly exists. Serious attempts will be made to develop urban health services
as per the recommendations of Krishnan Committee. Organic linkages will be forged with the urban development schemes including
Urban Basic Services for a comprehensive development ul' health and welfare services.
Local hospitals will be made responsible to run these centres and treat them as their extension counters for provid ing health services to
the community. Voluntary organisations and local bodies would be encouraged to develop partnership and ultimately tak ing full
responsibility for carrying out these programmes.
Health system research to develop a model of urban primary health care services will be undertaken.

Correct Answer. d

(45). National Institute of Epidemiology is located at :

a. New Delhi

b. Ahmedabad

c. Chennai

d. Kolkata

Solution. Ans-45: (c) Chennai Ref.: Read the text below Sol :
The National Institute of Epidemiology (NIE) was established on July 2, 1999 by merging the Central JALMA Institute Leprosy (CJIL),
Field Unit, Avadi with the Institute for Research in Medical Statistics (IRMS), Chennai. The broad objectives of the Institute cover
Development of human resources in epidemiology and bio-statistics, Networking of the various ICMR and non-ICMR Institutes at the
national level for epidemiological purposes, and Consultancy.
The Institute has the distinction of being the WHO Collaborating Centre for Epidemiology of Leprosy and identified as a Technical
Resource Group for Epidemiology of HIV by National AIDS Control Organizationm (NACO).
The Institute carries out a variety of research activities which include areas such as interventional studies, disease modeling, health
systems research, evaluation of health schemes and disease control programmes, issues of statistical methodology, epidemiological
investigations and clinical trials of traditional remedies.
The Institute is recognized by the Sree Chitra Tirunal Institute for Medical Sciences and Technology (Deemed University),
Thiruvananthapuram for the 2-year Field Epidemiology Training Programme (FETP-INDIA) leading to Master of Applied Epidemiology
(MAE) degree. The Institute has been conducting training programmes annually in bio-statistics, controlled clinical trials and basic
epidemiology for medical doctors, PG medical students and para-medical workers. It also conducts WHO-SEARO 10 day regional
workshops on Surveillance, Epidemic preparedness and Response periodically.

Correct Answer. c

Copyright 2014 Delhi Academy of Medical Sciences, All Rights Reserved. 26/86
(46). A new triage system was set up in an emergency unit of general hospital. To evaluate the new system, the waiting time of patients was
measured and compared with the waiting time at a comparable nearby hospital for a period of 6 months. The type of study (research
design) in above mentioned example is :

a. Cross-sectional study

b. Longitudinal study

c. Case control study

d. Clinical trial

Solution. Ans-46: (d) Clinical trial Ref.: Read the text below Sol :
For the most part, clinical trials have been concerned with evaluating therapeutic agents, mainly drugs.
Nevertheless, they are powerful tool and should be carried out before any new therapy, procedure or service is introduced.
Clearly in the question we are talking about a new triage system and before implementing it has to be tested first with a clinical trial.

Correct Answer. d

(47). Late expanding stage in demographic cycle is characterized by :

a. High birth rate and high death rate

b. Low birth and low death rate

c. Declining death rate and birth rate.

d. Unchanged birth rate, declining death rate

Solution. Ans 47: (c) Declining death rate and birth rate Ref P1 -349 Sol:

Correct Answer. c

(48). Regarding Prudent dietsalt intake should be reduced to an average of not more than:

a. 5gm/day

b. 7gm/day

c. 9gm/day

d. 10gm/day

Solution. Ans 48: (a) 5gm/day Reference P -461 Sol:


Salt intake should be reduced to an average of not more than 5gm/day.

Correct Answer. a

Copyright 2014 Delhi Academy of Medical Sciences, All Rights Reserved. 27/86
(49). Growth pattern of a population having annual growth rate of 1.0 to 1.5 is :

a. Slow growth

b. Moderate growth

c. Rapid growth

d. Very rapid growth

Solution. Ans 49: (c) Rapid growth Reference P -351 Sol:

Correct Answer. c

(50). The limiting aminoacid in soyabean is :

a. Methinonine

b. Lysine

c. Glutamine

d. Leucine

Solution. Ans 50: (c) Methinonine Reference P18 -454 Sol:


The proteins of soyabean are of relatively high nutritive value. The limiting amino acid is methionine

Correct Answer. c

(51). Lathyrism from khesari dal can be prevented by which process :

a. Parboiling

b. Filteration

c. Heating

d. Bleaching

Solution. Ans51: (a) Parboiling Reference P -468 Sol:


Parboiling is simple soaking in lime water overnight followed by boiling is credited to destroy the toxin.

Correct Answer. a

Copyright 2014 Delhi Academy of Medical Sciences, All Rights Reserved. 28/86
(52). True statement regarding small pox is : -

a. Rash predominant on flexor surfaces

b. Vesicles unilocular and umbilicated

c. Area of inflammation is seen around the vesicles

d. Scabs begins to form 10-14 days after the rash appears

Solution. Ans 52: (d) Scrabs begins to form 10-14 days after the rash appears Ref:Read the text below Sol: SMALL POX Incubation
period
About 12 days ( 7 17 ) days
Prodromal symptoms: Severe Distribution of rash
Palms and soles frequently involved
Axilla usually free
Rash predominant on extensor surfaces and bony prominences
Characteristics of rash
Deep seated
Vesicles multilocular and umbilicated
Only one stage of rash may be seen at one time
No area of inflammation is seen around the vesicles
Evolution of rash
Evolution of rash is slow,deliberate and majestic,passing through definite stages of macule,papule,vesicle and pustule
Scabs begins to form 10-14 days after the rash appears
Fever
Fever subsides with the appearance of rash,but may rise again in the pustular stage (secondary rise of fever)

Correct Answer. d

(53). Incorrect statement regarding Anganwadi worker is : -

a. There are about 100 anganwadi worker in each ICDS project

b. Part time worker.

c. Selected from community.

d. 1 for population of 10000

Solution. Ans 53: (d) 1 for population of 10000 Ref Read the text below Sol: Anganwadi worker
ICDS worker who is given training for 4 months and honorarium of rp 200-250 is given.
A worker is for population of 1000.
There are about 100 such workers in each ICDS project.
Services rendered by worker are health check up,immunization supplementary nutrition,health education,nor-formal pre-school
education .

Correct Answer. d

(54). Under micro-birth plan,mother should be informed about 4 Is.These Is includes all except : -

a. Identify the health centre for all referral.

b. Identify the mode of Delivery

c. Inform expected date of delivery

d. Inform dates of 3 ANC & TT Injection

Solution. Ans 54: (b) Identify the mode of Delivery Ref Read the text below Sol: Under micro-birth plan,mother should be informed
about 4 Is namely
Inform dates of 3 ANC & TT Injection (s) and ensure these are provided,
Identify the health centre for all referral,
Identify the Place of Delivery,
Inform expected date of delivery.

Correct Answer. b

Copyright 2014 Delhi Academy of Medical Sciences, All Rights Reserved. 29/86
(55). Under JANANI SURAKSHA YOJANA (JSY).In Low Performing States (LPS), cash assistance for institutional delivery is given upto : -

a. 1 live births

b. 2 live births

c. 3 live births

d. All births

Solution. Ans 55: (d) All births Reference Read the text below Sol: JANANI SURAKSHA YOJANA (JSY)
Janani Suraksha Yojana (JSY) is a safe motherhood intervention under the National Rural Health Mission (NRHM) being implemented
with the objective of reducing maternal and neo-natal mortality by promoting institutional delivery among the poor pregnant women.
JSY is a 100 % centrally sponsored scheme
The Yojana has identified ASHA, the accredited social health activist as an effective link between the Government and the poor
pregnant.
The scheme focuses on the poor pregnant woman with special dispensation for states having low institutional delivery rates namely the
states of Uttar Pradesh, Uttaranchal, Bihar, Jharkhand, Madhya Pradesh, Chhattisgarh, Assam, Rajasthan, Orissa and Jammu and
Kashmir. While these states have been named as Low Performing States (LPS), the remaining states have been named as High
performing States (HPS).
Limitations of Cash Assistance for Institutional Delivery:

Correct Answer. d

(56). Incorrect statement regarding Single-chamber incineration is : -

a. Good disinfection efficiency.

b. Significant emissions of atmospheric pollutants

c. The residues may be disposed of in landfills.

d. Need for highly trained operators

Solution. Ans 56: (d) Need for highly trained operators Reference P-647 Sol:

Correct Answer. d

Copyright 2014 Delhi Academy of Medical Sciences, All Rights Reserved. 30/86
(57). Colour monitors or labels curve put on vaccine bottles. The vaccine should be considered ineffective if : -

a. Colour of the square become blue like that of surrounding circle.

b. Colour of the square become red like that of surrounding circle.

c. Colour of the square become white like that of surrounding circle.

d. Colour of the square become yellow like that of surrounding circle.

Solution. Ans 57: (a) Colour of the square become blue like that of surrounding circle. Reference Read the text below Sol:
Colour monitors or labels curve put on vaccine bottles.
Each label has circle of deep blue Colour.
Inside it is a white square which changes colour and gradually become blue when vaccine bottle is exposed to higher temperature.
When the colour of the white square become blue like that of surrounding circle, the vaccine should be considered ineffective.

Correct Answer. a

(58). Disease not spread by Aedes mosquito : -

a. West nile fever.

b. Yellow fever.

c. Chikungunya fever

d. Rift Valley fever.

Solution. Ans 58: (a) Colour of the square become blue like that of surrounding circle. Ref Read the text below Sol: DISEASE SPREAD
BY AEDES MOSQUITO
Yellow fever
Dengue
Chikungunya fever
Rift Valley fever
filarial

Correct Answer. a

(59). Calculate median from the given values


20,21,22,23,24,25,26,27,28 : -

a. 21

b. 22

c. 23

d. 24

Solution. Ans 59: (d) 24. Reference Read the text below Sol:
Median is a statistical average which is calculated by arranging the data in ascending or descending order of magnitude and then the
value of the middle observation of the middle observation is located

Correct Answer. d

Copyright 2014 Delhi Academy of Medical Sciences, All Rights Reserved. 31/86
(60) Recommended dose of Vit A in Children>12 to prevent
. xerophthalmia is : -

a. 50000 I.U at birth

b. 100000 I.U once every 4-6 months

c. 2 lakh I.U once every 4-6 months

d. 3 lakh I.U within a month of giving birth

Solution. Ans 60: (c) 2 lakh I.U once every 4-6 months Reference
Read the text below Sol:

Correct Answer. c

(61). Gap in time between the onset of the primary case and the secondary case : -

a. Incubation period

b. Latent period

c. Serial interval

d. Generation period

Solution. Ans 61: (c) Serial interval Reference Read the text below Sol:
The serial interval is the gap in time between the onset of the primary case and the secondary case

Correct Answer. c

Copyright 2014 Delhi Academy of Medical Sciences, All Rights Reserved. 32/86
(62). All of the following are founding members of vision 2020 except-

a. WHO

b. UNICEF

c. ORBIS

d. International Agency For Prevention of Blindness

Solution. Ans-62: (b) UNICEF Ref: Read the text below Sol :
VISION 2020: The Right to Sight, is a global initiative to eliminate avoidable blindness by year 2020. The programme is a partnership between the
World Health Organization (WHO), and the International Agency for Prevention of Blindness (IAPB), a large umbrella organization for eye-care
professional groups and nongovernmental organizations (NGOs) involved in eye-care

Correct Answer. b

Copyright 2014 Delhi Academy of Medical Sciences, All Rights Reserved. 33/86
(63). Non-modifiable risk factor in CHD among the following :

a. Family history of the disease

b. Hypertension

c. Smoking

d. High carbohydrate diet

Solution. Ans-63: (a) Family history of the disease Ref: Read the text below Sol :

Correct Answer. a

(64). Multi-purpose worker scheme in India was introduced following the recommendation of :

a. Srivastava Committee

b. Bhore Committee

c. Kartar Singh Committee

d. Mudaliar Committee

Solution. Ans-64: (a) Srivastava Committee Ref: Read the text below Sol :
Kartar Singh Committee (1973) and Srivastava Committee (1975) recommended the provision of multipurpose workers in place of the
than unipurpose workers (such as ANM, basic health, worker, Vacinator, Family Panning Workers etc.) at each subcentre.
The Government of India implemented this recommendation in 1976, by providing at each subcentre 1 male health worker and 1 female
health worker, initially to cover a population of 8000 and in course of time 5000.

Correct Answer. a

(65) Under ICDS scheme; supplement nutrition for children between 1-6
. year is : -

a. 200 cal and 8-10gm protein

b. 300cal and 15 gm protein

c. 500 cal and 8-10gm protein

d. 500 cal and 25gm protein.

Solution. Ans 65: (b) 300cal and 15 gm protein Reference Read


the text below Sol: SUPPLEMENT NUTRITION UNDER ICDS
SCHEME
Children below 1 year 200 cal |8-10gm protein
Children between 1-6 year 300cal |15 gm protein
Pregnant women and nursing 500 cal |25gm protein

Correct Answer. b

Copyright 2014 Delhi Academy of Medical Sciences, All Rights Reserved. 34/86
(66). Regarding Schicks test positive test means : -

a. Red flush of 10-50 mm diameter on test arm

b. Red flush of 1-5 mm diameter on test arm

c. Red flush of 10-50 mm diameter on control arm

d. Red flush of 10-50 diameter on control arm.

Solution. Ans 66: (a) Red flush of 10-50 mm diameter on test arm Reference Read the text below Sol: O.2 ml of Schicks test toxin is
injected intradermally in to skin of forearm while in opposite arm is injected control.

Correct Answer. a

(67). Untrue statement regarding standard normal curve is: -

a. Total area of the curve is 1

b. Mean is 1

c. Standard deviation is 1

d. Mean median and made all coincide

Solution. Ans 67: (a) Mean is 1 Reference Read the text below Sol: STANDARD NORMAL CURVE
Is based on large no. of observation
It is smooth bell shaped, perfectly symmetrical curve it does not skew either to the right or left side
Total area of the curve is 1
Mean is O
Standard deviation is 1
Mean median and made all coincide

Correct Answer. a

(68). Among the following, which one is not a good dietary source of iron?

a. Liver

b. Jaggery

c. Fish

d. Milk

Solution. Ans-68: (d) Milk Ref: Park - 449. Sol :

Correct Answer. d

Copyright 2014 Delhi Academy of Medical Sciences, All Rights Reserved. 35/86
(69). The post-exposure prophylaxis against rabies requires a course of five doses of human diploid cell derived rabies vaccine. These shots are
given on :

a. Days 0, 1, 4, 10 and 30

b. Days 0, 3, 7, 14 and 30

c. Days 0, 7, 14, 21 and 28

d. Days 0, 4, 14, 24 and 30

Solution. Ans-69: (b) Days 0, 3, 7, 14 and 30 Ref: Read the text below Sol :
One dose of the vaccine should be administered on days 0, 3, 7, 14 and 30. All intramuscular injections must be given into the deltoid
region or, in small children, into the anterolateral area of the thigh muscle. Vaccine should never be administered in the gluteal region.

Correct Answer. b

(70). In normal curve

a. Mean = 2 standard : deviation

b. Mean = Median

c. Mean = Variance

d. Mean = 1 standard deviation

Solution. Ans-70: (b) Mean = Median Ref: Parks, 20th edn. Pg. 751, 752 Sol :

Correct Answer. b

(71). Biochemical oxygen demand is calculated to know

a. Organic waste

b. Inorganic waste

c. Total solids

d. Toxic substances

Solution. Ans-71: (a) Organic waste Ref: Parks-668 Sol :


Biochemical oxygen demand is the most important test done on sewage.
It is defined as the amount of oxygen absorbed by a sample of sewage during a specialized period, generally 5 days, at a specified
temperature, generally 20C for the aerobic destruction or use of organic matter by living organism.

Correct Answer. a

Copyright 2014 Delhi Academy of Medical Sciences, All Rights Reserved. 36/86
(72). PERT is a type of

a. Input output analysis

b. System analysis

c. Network analysis

d. Research technique

Solution. Ans-72: (c) Network analysis Ref: Parks- 775 Sol :


A network is a graphic plan of all events and activities to be completed in order to reach and end objective. It brings greater discipline in
planning.
The two common types of network analysis technique are :
Program evaluation and review technique (PERT)
Critical path method (CPM)

Correct Answer. c

(73). The number of stillbirths plus neonatal deaths per 1000 total births is called as:

a. Fetal mortality rate

b. Neonatal mortality rate

c. Perinatal mortality rate

d. Infant mortality rate

Solution. Ans-73: (c) Perinatal mortality rate Ref: Dutta - 605. Sol :
Perinatal mortality rate late fetal and dearly neonatal deaths weighting over 1000 g at birth expressed as a ratio per 1000 live births
weighting over 1000 g at birth.
Neonatal mortality rate is the number of neonatal death in a given year per 1000 live birth in that year.
Infant mortality rat the ratio of infant deaths registered in a given year to the total number of five births registered in the same year,
usually expressed as a rate per 1000 live births.

Correct Answer. c

Copyright 2014 Delhi Academy of Medical Sciences, All Rights Reserved. 37/86
(74). Which one of the following statements about vaccines is not correct :

a. Reconstituted measles vaccine should be used within one hour of reconstruction

b. DPT vaccine should be stored in the freezer compartment of the refrigerator

c. Diluent for measles vaccine should be stored in the lower shelf of the refrigerator

d. The diluents used for BCG vaccine is normal saline

Solution. Ans-74: (b) DPT vaccine should be stored in the freezer compartment of the refrigerator Ref: Read the text below Sol :
Recommended temperature for storage of vaccines :

Correct Answer. b

(75). In which one among the following Indian states is pellagra most prevalent ?

a. Kerala

b. West Bengal

c. Andhra Pradesh

d. Bihar

Solution. Ans-75: (c) Andhra Pradesh Ref: Park - 420 Sol :


Pellagra is a disease of poverty due to deficiency of niacin, characterized by 3 Ds diarrhea, dermatitis and dementia.
In addition, glossitis, stomatitis, depression, irritability, and delirium, while pellagra is historically a disease of maize-eating population,
it is still reported in India in the telangana area of Andhra Pradesh in Jowar (sorghum vulgare) eating population.
The excess of leucine is the cause of pellagra in both jowar and maize eaters.
Excess of leucine appears to interfere in conversion of tryptophan to niacin. Normally 60 mg of tryptophan is require to result in 1 mg
niacin.

Correct Answer. c

Copyright 2014 Delhi Academy of Medical Sciences, All Rights Reserved. 38/86
(76). When disease frequency is measured over a period of decades, it is called :

a. Cyclic trend

b. Seasonal trend

c. Secular trend

d. Periodic trend

Solution. Ans-76: (c) Secular trend Ref: Park - 60. Sol :


There is 3 types of time trends or fluctuations in disease occurrence
Short-term fluctuation
Period fluctuation (a) Seasonal trends in communicable disease, e.g. measles, varicella, cerebrospinal meningitis, upper respiratory
infections, malaria, (b) Cyclic trends occur in cycles spread over short periods of time due to naturally occurring variations in herd
immunity. As every 2 3 yrs measles in pre vaccination era. 6 9 yrs rubella. 7 10 yrs influenza.
Long term or secular trends changes in the occurrence of disease over a long period of time, generally several years or decades.
Example CHD, lung cancer and diabetes constantly shows upward trends in developed country of so followed by decline of tuberculosis,
typhoid fever, diphtheria and polio.

Correct Answer. c

(77). The protein quality indicator adopted by ICMR in recommending dietary protein requirements is :

a. Amino acid score

b. Net protein utilization

c. Biological value

d. Protein efficiency ratio

Solution. Ans-77: (b) Net protein utilization Ref: Read the text below Sol :
The net protein utilization, or NPU, is the ratio of amino acid converted to proteins to the ratio of amino acids supplied. This figure is
somewhat affected by the salvage of essential amino acids within the body, but is profoundly affected by the level of limiting amino acids
within a foodstuff.
Experimentally, this value can be determined by determining dietary protein intake and then measuring nitrogen excretion. One formula
for NPU is:
NPU = ((0.16 (24 hour protein intake in grams)) - ((24 hour urinary urea nitrogen) + 2) - (0.1 (ideal body weight in kilograms))) /
(0.16 (24 hour protein intake in grams)) As a value, NPU can range from 1 to 0, with a value of 1 indicating 100% utilization of dietary
nitrogen as protein and a value of 0 an indication that none of the nitrogen supplied was converted to protein.

Correct Answer. b

(78). Which fungicide is used as a spray for the control of bagassosis ?

a. 2% acetic acid

b. 2% formic acid

c. 2% propionic acid

d. 25% phenyl mercury

Solution. Ans-78: (c) 2% propionic acid Ref: Park JE. - 609-10 Sol :

Correct Answer. c

Copyright 2014 Delhi Academy of Medical Sciences, All Rights Reserved. 39/86
(79) What is the basic minimum illumination required for satisfactory vision
. ?

a. 10 15 foot candles

b. 15 20 foot candles

c. 20 25 foot candles

d. 25 30 foot candles

Solution. Ans-79: (b) 15 20 foot candles Ref: Park 549-50. Sol :

Correct Answer. b

(80). Eligibility criteria for sterilization include the following except :

a. Age of the female should be above 22 years

b. Age of the male should be below 60 years

c. Age of the female should not be more than 45 years

d. Couple should have at least two children

Solution. Ans-80: (b) Age of the male should be below 60 years Ref: Park -374 Sol :
Guideline for sterilization by government of India :
o Age of husband should not be less than 25 years and not be over 50 years.
o Age of the wife should not be less than 20 years or more than 45 years.
o Couple must have 2 living children
o If couple has 3 or more children, the lower limit of age of husband or wife may be relaxed.

Correct Answer. b

(81). Which one of the following is the correct statement? The positive health advocated by WHO implies that person should be able to express
as completely as possible the potentialities of his?

a. Physical freedom from sickness

b. Preventive, curative and primitive health

c. Genetic heritage

d. Physical, mental and social well being

Solution. Ans-81: (d) Physical, mental and social well being Ref: Park - 15. Sol :
Positive health (WHO) A state of physical, mental and social well being.

Correct Answer. d

Copyright 2014 Delhi Academy of Medical Sciences, All Rights Reserved. 40/86
(82). The pattern of disease in a community described in terms of the important factors which influence its occurrence is known as :

a. Epidemiology

b. Confounding factors

c. Iceberg of disease

d. Community diagnosis

Solution. Ans-82: (d) Community diagnosis Ref: Park - 43. Sol :


Community diagnosis The pattern of disease in a community described in terms of the important factors which influence this pattern.

Correct Answer. d

(83). Census is conducted in very years in India

a. 25

b. 15

c. 10

d. 20

Solution. Ans-83: (c) 10 Ref: Parks- 743 Sol :


Census is conducted every 10 years in India. Started in 1881.
The last census was held in March 2001.
It is usually conducted at the end of the first quarter of the first year in each decade.
Current population of India as obtained from the internet is about 1.15 billion. It has increased more than 3 times since independence.

Correct Answer. c

(84). Which one is not true of case fatality rate ?

a. It is a ratio

b. Time interval is non-specified

c. It may vary from the same disease in different epidemics

d. It is useful in chronic diseases

Solution. Ans-84: (d) It is useful in chronic diseases Ref: Parks- 54 Sol :


Case-fatality rateUsually expressed as the percentage of persons diagnosed as having a specified disease who die as a result of that
illness within a given period.
This term is most frequently applied to a specific outbreak of acute disease in which all patients have been followed for an adequate
period of time to include all attributable deaths.
The case-fatality rate must be clearly differentiated from the mortality rate (q.v.). (Synonyms: fatality rate, fatality percent-age, case-
fatality ratio)

Correct Answer. d

Copyright 2014 Delhi Academy of Medical Sciences, All Rights Reserved. 41/86
(85). All of the following are determinants for the essential components to calculate Physical Quality of Index (PQLI) except

a. Infant Mortality Rate

b. Life expectancy at age one year

c. Basic literacy rate in population

d. Life expectancy at birth

Solution. Ans-85: (d) Life expectancy at birth Ref: Parks- 16 Sol :


The Physical Quality of Life Index (PQLI) is an attempt to measure the quality of life or well-being of a country.
The value is the average of three statistics: basic literacy rate, infant mortality, and life expectancy at age one, all equally weighted on a
0 to 100 scale.

Correct Answer. d

(86). MTP Act of 1971 provides the following indications except

a. Where continuation of pregnancy endangers mothers life

b. Where pregnancy is a result of Rape

c. When acceptors requires incentives

d. Failure of contraceptive device

Solution. Ans-86: (c) When acceptors requires incentives Ref: Parks- 434 Sol :
The conditions under which pregnancy may be terminated under the MTP act (1971) include :
1. Medical : Where continuation of pregnancy endangers mothers life. 2. Eugenic : Where there is a high risk of the child being born
with serious mental/physical handicaps. 3. Humanitarian : When pregnancy is a result of rape. 4. Failure of contraceptive device. 5.
Socio-economic : Extremely poor and totally unreasonable socioeconomic conditions leading to injury to child or mother.

Correct Answer. c

Copyright 2014 Delhi Academy of Medical Sciences, All Rights Reserved. 42/86
(87). The daily diet of an adult sedentary male worker consists of :
Cereals 460 grams
Pulses 40 grams
Vegetables, roots and tubers 150 grams
Milk 100 grams
Oils and fats 40 grams
Sugar and jiggery 30 grams
In the context of balanced vegetarian diet, the diet of the worker mentioned above is deficient in terms of the recommended quantity in respect
of :

a. Cereals

b. Milk

c. Pulses

d. Vegetables, roots and tubers

Solution. Ans-87: (b) Milk Ref.: Read the text below Sol :
Balanced diet (gm) :

Correct Answer. b

(88). As per nutritional surveys, the diet of an average Indian of lower socioeconomic strata is highly deficient in :

a. Thiamin

b. Riboflavin

c. Nicotinic acid

d. Vitamin C

Solution. Ans-88: (b) Riboflavin Ref.: Park - 420. Sol :


Studies conducted at National Institute of Nutrition in India showed that subclinical riboflavin deficiency (as judged by erythrocyte
glutathione reductase activation test) was present in over 80% of low income group children and adults.
RDA Thiamin (0.5 mg/1000 Kcal), riboflavin 0.6 mg/1000 kcal, niacin 6.6 mg/1000 kcal.

Correct Answer. b

Copyright 2014 Delhi Academy of Medical Sciences, All Rights Reserved. 43/86
(89). Which of the following should be stored in a deep freezer in order to preserve their potency ?
(1) BCG vaccine (2) Oral polio vaccine (3) Measles vaccine (4) DPT vaccine
Select the correct answer using the codes given below :
Codes :

a. 1, 2 and 3

b. 2, 3 and 4

c. 1 and 4

d. 1, 2 ; 3 and 4

Solution. Ans-89: (a) 1, 2 and 3 Ref.: Park - 93. Sol :


Order in which vaccine lose their potency when exposed to heat are as follows
1. Polio 2. Measles 3. BCG 4. DPT 5. DT 6. Tetanus

Correct Answer. a

(90). The Pear index indicates the number of accidental pregnancies per :

a. 1000 population

b. 100 live births

c. 100 women in the age group of 15 to 44 years

d. 100 women years

Solution. Ans-90: (d) 100 women years Ref.: Park - 375 Sol :
Pearl indexThe failure rate of any contraceptive is calculated in terms of pregnancy rate per hundred women years (HWY) of use.

Correct Answer. d

(91). Following the introduction of multidrug therapy, for the purposes of estimating the prevalence of leprosy, a case is defined as :

a. A person showing clinical signs of the disease with or without bacteriological confirmation

b. Only bacteriologically confirmed patient

c. Only smear negative intermediate type

d. Lepromatous and borderline patient

Solution. Ans-91: (a) A person showing clinical signs of the disease with or without bacteriological confirmation Ref.: Park -248 Sol :
Following the introduction of multidrug therapy, following definitions have been proposed by WHO
1. Case of leprosy A person showing clinical signs of the disease with or without bacteriological confirmation of the diagnosis. This
definition is for estimating the prevalence of leprosy. 2. Paucibacillary leprosy Only smear negative intermediate (I), tuberculoid (T),
borderline tuberculoid (BT) and pure neural (N) in Indian classification. In India, 60% of total leprosy is paucibacillary. 3. Mutibacillary
leprosy Lepromatous (L), borderline lepromatous (BL) and all smear positive. 4. Adequate treatment (a) For paucibacillary
received 6 monthly doses of combined therapy within 9 months, (b) Multibacillary cases patient has received 24 monthly doses of
combined therapy within 36 months. 5. Regular treatment Has received combined therapy for at least two-thirds of months in any
interval of time.

Correct Answer. a

Copyright 2014 Delhi Academy of Medical Sciences, All Rights Reserved. 44/86
(92). Which one of the following strategies is aimed at AIDS control in India:

a. Detection and treatment of AIDS cases in the community

b. Detection and isolation of HIV infected persons in the community

c. Immunoprophylaxis of the risk groups

d. Community education towards behavioural change

Solution. Ans-92: (d) Community education towards behavioural change Ref.: Park - 337 Sol :
The Health Belief Model (HBM) is a psychological model that attempts to explain and predict health behaviors by focusing on the
attitudes and beliefs of individuals.
The HBM was developed social psychologists in the United States Public Health Service to explain the lack of public participation in
health screening and prevention programs (e.g., a free and conveniently located tuberculosis screening project).
Since then, the HBM has been adapted to explore a variety of long- and short-term health behaviors, including sexual risk behaviors and
the transmission of HIV/AIDS.

Correct Answer. d

(93). Consider the following data :


Mid-year population of a town 16,80,000
Number of live births 67,315
Number of deaths 31,410
Number of deaths under one year 17,315
Number of deaths under 28 days 9,231
Based on this data, the post-neonatal mortality rate will be :

a. 100/1000

b. 110/1000

c. 120/1000

d. 130/1000

Solution. Ans-93: (c) 120/1000 Ref.: Park - 576 Sol :

Neonatal mortality rate = No. of neonatal deaths in a given year per 1000 live births in that year.
Infant mortality rate ratio of infant deaths registered in a given year to the total no. of live births registered in the same year, usually
expressed as rate per 1000 live births.

Correct Answer. c

Copyright 2014 Delhi Academy of Medical Sciences, All Rights Reserved. 45/86
(94). Biological transmission of microfilaria in mosquito is :

a. Cyclodevelopmental

b. Cyclopropagative

c. Propagative

d. Transovarian

Solution. Ans-94: (a) Cyclodevelopmental Ref.: Park - 574 Sol :


Biological transmission :
o Propagative Disease agent undergoes no cyclical change but multiplies in the body of the vector, e.g. plague bacilli in rat fleas.
o Cyclo-propagative Disease agent undergoes cyclical change and multiplies in the body of the arthropod, e.g. malaria parasite in
anopheline mosquito.
o Cyclo-development When disease agent undergoes cyclical change but does not multiply in the body of arthropod, e.g. filarial parasite
in culex and guineaworm embryo in cyclops.

Correct Answer. a

(95). All of the following facts are true with group discussion except

a. Group discussion is very effective method of health education

b. Group members should not have known each other before

c. The group should sit in a circle

d. There should be a group leader to initiate

Solution. Ans-95: (b) Group members should not have known each other before Ref: Parks- 767 Sol :
For effective group discussion, the group should comprise not less than 6-12 members. The participants are seated in a circle, so that
each is fully visible to all the others.
There should be a group leader to initiate the discussion, encourage everyone to participate and sums up the discussion in the end.
Group discussion is effective if the members know each other beforehand so that they can discuss their views freely.

Correct Answer. b

(96). Habits and customs are conducive to cancer as evident below except

a. Kangri cancer in Kashmir due to hot pot in winter

b. Oral cancer due to pan chewing in India

c. Penile cancer and cervical cancer following circumcision

d. Lung cancer due to smoking

Solution. Ans-96: (c) Penile cancer and cervical cancer following circumcision Ref: Parks- 334 Sol : Customs, habits and lifestyles
conducive to cancer are
Lung cancer due to smoking
Tobacco and betel chewing and oral cancer
Kangri cancer in Kashmir due to carrying hot pots in winter
Cervical cancer and penile cancer are not due to circumcision.
Cervical cancer is due to Human Papilloma Virus (HPV) and circumcision protects against penile cancer.

Correct Answer. c

Copyright 2014 Delhi Academy of Medical Sciences, All Rights Reserved. 46/86
(97). Maternal mortality rate (MMR) is defined as number of maternal deaths per

a. 1000 live births

b. 1,00,000 live births

c. 10,000 live births

d. 100 live births

Solution. Ans-97: (a) 1000 live births Ref: Parks- 479 Sol :
Maternal mortality rate (MMR) measures the risk of women dying from puerperal causes and is defined as (total number of female
deaths due to complications of pregnancy, childbirth or within 42 days of delivery from puerperal causes in an area during a given year)
/ (total number of live births in the same area and year) 1000.
Ideally the denominator should include all deliveries and abortions.
The MMR should be expressed as a rate per 1000 live births. In developed countries, MMP has declined significantly ; hence they use
the multiplying factor as 1,00,000 to avoid fractions in calculating MMR.

Correct Answer. a

(98). Generation time in epidemiology is defined as

a. The interval between marriage and the birth of first child

b. The interval of time between the receipt of infection by host and maximal infectivity of the host

c. The interval of time between primary case and secondary cases

d. Interval of time between invasion by infection agent and appearance of first sign or symptom of the disease/in question

Solution. Ans-98: (b) The interval of time between the receipt of infection by host and maximal infectivity of the host Ref: Parks-95 Sol :
Generation time is defined as the interval of time between the receipt of infection by host and maximal infectivity of the host.

Correct Answer. b

(99). The overall prevalence of tuberculosis infection in India as per 4th round of longitudinal survey was

a. 20 %

b. 30%

c. 40%

d. 50%

Solution. Ans-99: (b) 30% Ref: Parks- 160 Sol :


The prevalence of infection as reported by the National Tuberculosis Institute, Bangalore in their longitudinal survey 4th round was
about 30%

Correct Answer. b

Copyright 2014 Delhi Academy of Medical Sciences, All Rights Reserved. 47/86
(100). For an adult Indian male the daily requirement of protein is expressed as

a. 0.5 g/kg body weight

b. 0.75 g/kg body weight

c. 1 gm/kg body weight

d. 1.50 g/kg body weight

Solution. Ans-100: (c) 1 gm/kg body weight Ref: Parks- 549 Sol :
The Indian council of Medical Research (ICMR) in 1981 recommended 1g protein/kg body weight for an Indian adult assuming a NPU of
65 for dietary proteins.

Correct Answer. c

(101). Lathyrism is due to consumption of

a. Red gram dhal

b. Contaminated ground nuts

c. Bengal gram dhal

d. Khesari dhal

Solution. Ans-101: (d) Khesari dhal Ref: Parks- 558 Sol :


Neurolathyrism is a crippling disease of the nervous system and is characterized by gradually developing spastic paralysis of limbs
occurring mostly in adults consuming the pulse Lathyrus sativus (kesari dhal) in large quantities.

Correct Answer. d

(102). An upper limit of noise which people can tolerate without sub damage to their hearing is

a. 45 db

b. 65 db

c. 85 db

d. 105 db

Solution. Ans-102: (c) 85 db Ref: Parks- 648 Sol :


A daily exposure of upto 85 dB is about the limit people can tolerate without substantial damage to their hearing, i.e. recommended
maximum is 85 dB.
Exposure to noise above 160 dB may rupture the tympanic membrane.

Correct Answer. c

(103). The mean radiant temperature is measured by

a. Dry bulb thermometer

b. Wet bulb thermometer

c. Sixs maximum and minimum thermometer

d. Globe thermometer

Solution. Ans-103: (d) Globe thermometer Ref: Parks-653 Sol :


The globe thermometer is used for the direct measurement of the mean radiant temperature of surroundings.
The difference between the globe thermometer temperature and that of the ordinary dry bulb is a measure of the radiant heat.

Correct Answer. d

Copyright 2014 Delhi Academy of Medical Sciences, All Rights Reserved. 48/86
(104). Pulse polio immunization is administration of OPV to

a. All children between 0-5 years of age on a single day, irrespective of their previous immunization status

b. Children in the age group of 0-1 year only who have not been immunized earlier

c. Children in the age group of 12-24 months only as the booster dose

d. All children between 0-5 years of age, whenever there is an outbreak of poliomyelitis

Solution. Ans-104: (a) All children between 0-5 years of age on a single day, irrespective of their previous immunization status Ref:
Parks-183, 378 Sol :
The pulse polio immunization (PPI) was started in 1995, 9th of December and followed up 6 weeks later on 20th January 1996. The first
PPI targeted children under the age of 3 years irrespective of their immunization status.
The term pulse has been used to describe this sudden simultaneous mass administration of OPV on a single day to all children 0-5
years of age, regardless to previous immunization.
The dose of OPV during PPIs are extra doses and do not replace the doses received during routine immunization services.

Correct Answer. a

(105). National Health Policy recommends setting up of a sub-center in desert and hilly areas for a population of

a. 1000

b. 2500

c. 3000

d. 5000

Solution. Ans-105: (c) 3000 Ref: Parks- 805, 803 Sol :


National Health Policy recommends setting up of a sub-center in desert and hilly areas for a population of 3000.

Correct Answer. c

(106). Minimum microbiological criteria for water quality includes

a. All samples should be free of coliforms

b. Not more than 5% samples should have any coliform

c. Any 100 ml should not have more than 3 E. coli

d. Coliforms should not be detectable in 3 consecutive samples of 100 ml of water

Solution. Ans-106: (a) All samples should be free of coliforms Ref: Parks- 631 Sol :
In all water intended for drinking, E. coli or thermotoleranat coliform bacteria must not be detectable in any 100 ml sample.

Correct Answer. a

Copyright 2014 Delhi Academy of Medical Sciences, All Rights Reserved. 49/86
(107). Additional daily energy requirement for a lactating woman with a two month old child will be

a. 450 calories

b. 550 calories

c. 650 calories

d. 750 calories

Solution. Ans-107: (b) 550 calories Ref: Parks- 548 Sol :


The energy requirements of women are increased by
Pregnancy (+ 300 kCal daily throughout pregnancy ) and
Lactation (+ 550 kCal adily during the first 6 months, and + 400 kCal daily during the next 6 months.

Correct Answer. b

(108). In a study of relationship between obesity and cardiovascular disease, increased cholesterol and hyperglycemia are.

a. Confounders

b. Intervening variables

c. Direct causes

d. Independent variables

Solution. Ans-108: (a) Confounders Ref: Park - 66. Sol :


Confounding factor : One which is associated both with exposure and disease and is distributed unequally in study and control group.
Although associated with exposure under investigation, is itself, independently of any association, a risk factor for disease.

Correct Answer. a

(109). The term disease control describes ongoing operation saimed at reducing the following, except:

a. Incidence of disease

b. Financial burden to the community

c. Virulence of the disease agent

d. Disease duration

Solution. Ans-109: (c) Virulence of the disease agent Ref: Park - 35. Sol :
The term disease control describes ongoing operations aimed at reducing :
o The incidence of disease.
o The duration of disease, and consequently the risk of transmission.
o The effects of infection, including both the physical and psychosocial complication.
o The financial burden to the community.

Correct Answer. c

Copyright 2014 Delhi Academy of Medical Sciences, All Rights Reserved. 50/86
(110). Kappuswamy index includes all except: -

a. Income

b. Education

c. Occupation

d. Residential Address

Solution. Ans 110: (d) Residential Address Reference Read the text below Sol:

Correct Answer. d

(111). For which one of the following has the cytotoxic effect of vitamin on human lymphocytes in vitro at high concentration been reported ?

a. Vitamin A

b. Vitamin D

c. Vitamin E

d. Vitamin K

Solution. Ans-111: (c) Vitamin E Ref: Read the text below Sol :
The possible mechanism of cytotoxication of vitamin E short-chain derivatives in cell cultures is offered, namely: their high permeability
through a plasmatic membrane allows to create inside the cells a concentration, sufficient for detergent-like rupture of the lysosomal
membranes, that results in the entering of lysosomal enzymes in cytosol and destruction of cells by necrosis

Correct Answer. c

Copyright 2014 Delhi Academy of Medical Sciences, All Rights Reserved. 51/86
(112). Which one of the following cooking oils contains highest percentage of linoleic acid ?

a. Safflower oil

b. Sunflower oil

c. Corn oil

d. Groundnut oil

Solution. Ans-112: (a) Safflower oil Ref: Park JE. Textbook of Preventive and Social Medicine, (18th Ed.), Pg. 440. Sol :

Correct Answer. a

(113). Hookworm will be considered Minor public health problem if Chandler index is : -

a. Below 200

b. 200-250

c. 260-300

d. Above 300

Solution. Ans 113: (c) 260-300 Reference Read the text below Sol: Chandler index Average no. of hookworm eggs|gm of faeces for the
entire community

Correct Answer. c

Copyright 2014 Delhi Academy of Medical Sciences, All Rights Reserved. 52/86
(114). Point source epidemic is characterized by all of the following except: -

a. Common source.

b. Single exposoure.

c. The epidemic curve rises and falls rapidly

d. The epidemic curve does not have more than 1 secondary waves

Solution. Ans 114: (d) The epidemic curve does not have more than 1 secondary waves. Ref Read the text below Sol: Point source
epidemic
Common source
Single exposoure
All cases develop within 1 incubation period disease
The epidemic curve rises and falls rapidly with no secondary waves

Correct Answer. d

(115). Incubation period of Rubella is : -

a. 7 days

b. 2-6 days

c. 2-3 wks

d. 10 days from exposure to onset of fever and 14 days to appearance of rash

Solution. Ans 115: (c) 2-3 wks. Reference Read the text below Sol:

Correct Answer. c

(116). Incidence is derived from ?.

a. Cross sectional

b. Case and control

c. Prospective study

d. Retrospective study

Solution. Ans 116: (c) Prospective study Ref: Park- 74 Sol:


Incidence total no of new cases occurring in a defined population during specified period of time.
Cross-sectional studies give an estimate of prevalence, while case-control is an example of retrospective study and both give an estimate
of the relative risk, known as Odds ratio.
Incidence the no. of new cases occurring in a defined population during specified period of time.
Here period is usually one year
Special incidence rates Attack rate, sec attack rate, hospital admission ratio
Incidence is derived during epidemiology study Cohort study

Correct Answer. c

Copyright 2014 Delhi Academy of Medical Sciences, All Rights Reserved. 53/86
(117). All are true about cluster sampling except?

a. Sample size based on hypothesis

b. Rapid assessment

c. Samples provide equal type of various clusters

d. Easy

Solution. Ans 117: (a) Sample size based on hypothesis Ref: Read the text below Sol:
Choice a seems to be the right answer as the sample size is not based any hypothesis but then again option c is not clear.
The clusters are not equal in cluster sampling technique and the meaning of equal is not mentioned as in what sense. Still I would go
with option a as the correct answer.

Correct Answer. a

(118). False + is more with screening test with?

a. Decreased prevalence

b. Increased prevalence

c. Increased sensitivity

d. Decreased specificity

Solution. Ans 118: (d) Decreased specificity Ref: Read the text below Sol:
Decreased specifity means increased number of false positives.
Specifity is the ability of a test to correctly identify the true negatives out of all disease negatives.

Correct Answer. d

(119). Given Logo is related with ?

a. Swine Flu

b. Dengue

c. Japanese encephalitis

d. Malaria

Solution. Ans 119: (a) Swine Flu Ref Read the text below Sol:
Swine influenza, also called pig influenza, swine flu, hog flu and pig flu, is an infection caused by any one of several types of swine
influenza viruses. Swine influenza virus (SIV) or swine-origin influenza virus (S-OIV) is any strain of the influenza family of viruses that is
endemic in pigs.[2] As of 2009, the known SIV strains include influenza C and the subtypes of influenza A known as H1N1, H1N2, H2N1,
H3N1, H3N2, and H2N3.
Swine influenza virus is common throughout pig populations worldwide. Transmission of the virus from pigs to humans is not common
and does not always lead to human flu, often resulting only in the production of antibodies in the blood. If transmission does cause
human flu, it is called zoonotic swine flu. People with regular exposure to pigs are at increased risk of swine flu infection.

Correct Answer. a

Copyright 2014 Delhi Academy of Medical Sciences, All Rights Reserved. 54/86
(120). Fish is a poor source of :

a. Fe

b. Iodine

c. Phosphorus

d. Vitamin A

Solution. Ans120-C Phosphorus Ref: Read the text below Sol:


A tricky question. In fact, fish is a rich source of all the above mentioned nutrients. However it is exceptionally good source of Iodine and
Vitamin A.
Of the remaining two choices, Phosphorus seems to be the better answer as it is in same quantities in fish as other meat products.
Fish Protein rich (15-25%) Rich in
Unsaturated fatty acids
Vit A also remember fish oil richest natural source of Vit A (retinol)
Vit D

Correct Answer. c

(121). Seventh disability is defined in part of article of disabilities act of 1955 includes

a. Drug addiction

b. Disability by accident

c. Mental illness

d. Neurological illness

Solution. Ans 121: (c) Mental illness Ref:http://www.indiatogether.org/health/opinions/jabidi2.htm,


http://www.hinduonnet.com/mag/2003/11/30/stories/2003113000240400.htm, http://socialwelfare.delhigovt.nic.in/disabilityact.htm Sol:
The Persons with Disabilities (Equal Opportunities, Protection of Rights and Full Participation) Act was passed by both the Houses of our
Parliament in a single day in December 1995. The law was notified on 7th February 1996. The Disability Act of 1995 defines 'disability' to
mean
Blindness;
Low vision;
Leprosy-cured;
Hearing impairment;
Locomotor disability;
Mental retardation, and
Mental illness.
The Disability Act of 1995 not only included mental illness under the definition of 'disability' but also mandates that its Central
Coordination Committee (which has representation at the 'Secretary to the Government of India' level of the Health Ministry!) will: (1)
review and coordinate the activities of all Governmental and Non-Governmental Organizations which are dealing with matters relating to
persons with disabilities; (2) develop a national policy to address issues faced by persons with disabilities; (3) advise on the formulation
of policies, programmes, legislations and projects with respect to disability.
In a tragic incident at Erwadi in Tamilnadu during August 2001, 25 people, including 11 women, were charred to death. A devastating
fire broke out at 5 a.m. in the thatched hostel housing them. Out of the 46 hostel inmates, 40 were chained to their beds.
They kept screaming for help but no one came to their rescue. The 46 hostel inmates were mentally ill. The Erwadi tragedy of Aug 2001
attracted international attention. That it happened is indicative of a deep-rooted apathy towards people with mental illness.

Correct Answer. c

Copyright 2014 Delhi Academy of Medical Sciences, All Rights Reserved. 55/86
(122). Daily required dose of iron in an adult man is:

a. 0.7mg

b. 0.58 mg

c. 0.9 mg

d. 1.0 mg

Solution. Ans-122: (c) 0.9 mg Ref.: Read the text below Sol : Requirement of iron :
0.7 mg is requirement for infants (5-12 months) and post menopausal woman
0.8 mg is required for first half of pregnancy
0.9 mg is the amount required by an adult male
mg is required by children (1-12 years)

Correct Answer. c

(123). Amount of proteins in human milk (in gms) :

a. 1.1

b. 2.2

c. 3.3

d. 4.4

Solution. Ans.-123: (a) 1.1 Ref.: Read the text below Sol : Each 100 grams of breast milk yields approximately :
65 Kilocalories
88 g water
7.4 g carbohydrates (primarily lactose)
3.4 g fat
g protein

Correct Answer. a

(124). Best indicator for monitoring the effectiveness of iodine deficiency control program me is :

a. Neonatal hypothyroidism

b. Hypothyroidism in male population

c. Hypothyroidism in female population

d. Size of goitrous lesion

Solution. Ans-124: (a) Neonatal hypothyroidism Ref.: Read the text below Sol :
Neonatal hypothyroidism is a sensitive pointer to environmental iodine deficiency and is thus an effective indicator for monitoring the
impact of the iodine deficiency control programme.

Correct Answer. a

Copyright 2014 Delhi Academy of Medical Sciences, All Rights Reserved. 56/86
(125). How many gram of protein is given in mid day meal scheme :

a. 4-8 gram

b. 8-12 gram

c. 12-16 gram

d. 16-20 gram

Solution. Ans-125: (b) 8-12 gram Ref.: Read the text below Sol :

Correct Answer. b

(126). Under National Cancer control programme, oncology wings were sanctioned to :

a. Regional cancer institutes

b. District hospitals

c. Medical college hospitals

d. Voluntary agencies treating cancer patients

Solution. Ans-126: (c) Medical college hospitals Ref.: Park JE-346 Sol : NATIONAL CANCER CONTROL PROGRAMME 1975-76
New schemes have been initiated from 1990-91
Scheme for district projects
Development of oncology wings in medical colleges/hospitals
Scheme for financial assistance to voluntary organizations

Correct Answer. c

(127). Exposure to 90 db and 4000 Hz noise results in :

a. Auditory fatigue

b. Temporary hearing loss

c. Permanent hearing loss

d. Rupture of tympanic membrane

Solution. Ans-127: (a) Auditory fatigue Ref.: Park --551 Sol :

Correct Answer. a

Copyright 2014 Delhi Academy of Medical Sciences, All Rights Reserved. 57/86
(128). Which one of the following will be affected by interobserve variation in epidemiological studies?

a. Sensitivity

b. Predictive value of the positive test

c. Specificity

d. Reliability

Solution. Ans-128: (d) Reliability Ref.: Park -115 Sol : CRITERIA MUST SATISFY THE SCREENING TEST :
Acceptability
Repeatability (reliability, precision, reproducibility) repeatability of test depends upon
Observer variation
Intraobserver variation (within observer) different results of two measurements, (e.g. BP) in the same subject, at same time by a single
observer.
Interobserver (between observer) variations between different observer on the same subject or material.
Tested by repeat measurement at same times
Biological (subject) variation due to
Changes in the parameters observed
Variation in the way patients perceive their symptoms and answer
Regression to the mean
Tested by repeat measurement over time
Errors relating to technical methods
Validity (accuracy) refers to what extent the test accurately measures which it purports to measure
Two component sensitivity and specificity

Correct Answer. d

(129). Which one of the following is a characteristic of epidemiology of leishmaniasis in India?

a. It does not occur at high altitudes

b. Incubation period is 3-8 days

c. Females are more affected

d. It is not a fatal disease

Solution. Ans-129: (a) It does not occur at high altitudes Ref.: Park--257 Sol :
Kala azar is mostly confined to the plain area. It does not occur above 2000 feet (600 meters).
Males affected twice than females. While in India peak agge group is 5 to 9 years. It affects poorest to poor socioeconomic group.
Incubation period is quiet variable from month to year; in man is generally 1 to 4 months, range 10 days to 2 years. Extrinsic incubation
period (after an infective blood meal sandfly become infective) is 6 to 9 days.
Disease is fetal if not treated properly.
Vector P. argentipes is vacctor of kala azar
P. Papatasi and P. sergenti are vectors of cutaneous leimaniasis.

Correct Answer. a

Copyright 2014 Delhi Academy of Medical Sciences, All Rights Reserved. 58/86
(130). Which one of the following is the correct chronological order in which the given Acts were passed in India?

a. Quarantine Act, Vaccination Act, Sarda Act, Factory Act

b. Quarantine Act, Sarda Act, Factory Act, Vaccination Act

c. Sarda Act, Vaccination Act, Quarantine Act, Factory Act

d. Vaccination Act, Quarantine Act, Factory Act, Sarda Act

Solution. Ans-130: (a) Quarantine Act, Vaccination Act, Sarda Act, Factory Act Ref.: Read the text below Sol :
The Quarantine Act was promulgated in 1825
The Vaccination Act was passed in 1880
The Child Marriage Restrant Act (Sarda Act) came into effect fixing the minimum age of marriage at 14 for girls and 18 for boys in 1930
The Indian Factory Act of 1948 was amended in 1976. Although the Indian Factory Act was passed in 1881. But among above choice the
nearest answer is (a) so we can go through choice (a).

Correct Answer. a

(131). Which of the following is false regarding acrodermatitis enteropathica:

a. Low serum zinc levels

b. Triad of diarrhea, dementia and dermatitis

c. Autosomal recessive condition

d. None of the above

Solution. Ans-131: (b) Triad of diarrhea, dementia and dermatitis Ref.: Read the text below Sol :
Acrodermatitis enteropathica is an autosomal recessive metabolic disorder affecting the uptake of zinc, characterized by periorificial
(around the natural orifices) and acral (in the limbs) dermatitis, alopecia (loss of hair), and diarrhea.
Similar features may be present in acquired zinc deficiency. This disease also is related to deficiency of zinc due to congenital causes.

Correct Answer. b

(132). If in a community, a high proportion of newborn babies suffers from methaemoglobinaemia, then which one of the following inferences
would be most relevant :

a. The community is exposed to industrial smoke

b. The local water supply is very rich in nitrites

c. The babies are suffering from the effects of mercury poisoning

d. The haemoglobin of the babies blood is in a very reduced state

Solution. Ans-132: (b) The local water supply is very rich in nitrites Ref.: Park - 504 Sol :
The guideline value for nitrate in drinking water is solely to prevent methamoglobinemia, which depends upon the conversion of nitrate
into nitrite.
Bottle-fed infants of less than 3 months of age are most susceptible.

Correct Answer. b

Copyright 2014 Delhi Academy of Medical Sciences, All Rights Reserved. 59/86
(133). Following index measures average achievements in basic dimensions of human development.

a. Human poverty index

b. Human development index

c. Gender empowerment index

d. Life expectancy at birth

Solution. Ans-133: (b) Human development index Ref:Park19-16 Sol:


Human development index (HDI) is defined as a composite index combining indicators representing three dimensions longevity (life
expectancy at birth) : knowledge (adult literacy rate and mean years of schooling) : and income (real GDP per capita in purchasing power
parity in US dollars) The HDI values ranges between 0 to 1.
The HDI value for a country shows the distance that it has already travelled comparisons with other countries.
To construct the index, fixed MINIMUM and MAXIMUM values have been established for each of these indicators.
Life expectancy at birth : 25 years and 85 years. Adult literacy rate : 0 per cent and 100 percent Combined gross enrolment ratio : 0 per
$ 100 and $ 40,000 (PPP$)

Correct Answer. b

(134). Iceberg phenomenon differentiates

a. Apparent & inapparent

b. Symptomatic & asymptomatic

c. Cases & carriers

d. Diagnosed and undiagnosed

Solution. Ans-134: (a) Apparent & inapparent Ref:Park-36 Sol:


A concept closely related to the spectrum of disease is the concept of the iceberg phenomenon of disease.
According to this concept, disease in a community may be compared with an iceberg.
The floating tip of the iceberg represents what the physician sees in the community. i.e. clinical cases.
The vast submerged portion of the iceberg represents the hidden mass of disease, i.e., latent, inapparent, presymptomatic and
undiagnosed cases and carriers in the community.
The waterline represents the demarcation between apparent and inapparent disease.

Correct Answer. a

(135). Seasonal trends is shown by

a. Varicella

b. Polimyelitis

c. Malaria

d. All

Solution. Ans-135: (d) All Ref:Park-60 Sol: SEASONAL TREND


Seasonal variation is a well-known characteristic of many communicable diseases, e.g. measles, varicella, cerebro-spinal meningitis,
upper respiratory infections, malaria, etc.
For example. measles is usually at its height in early spring and so is varicella.
Upper respiratory infections frequently show a seasonal rise during winter months.
Gastrointestinal infections are prominent in summer months because of warm weather and rapid multiplication of flies.

Correct Answer. d

Copyright 2014 Delhi Academy of Medical Sciences, All Rights Reserved. 60/86
(136). Life long immunity is seen in all of the following except

a. Pertussis

b. Mumps

c. Rubella

d. None

Solution. Ans-136: (a) Pertussis Ref:Read the text below Sol:


Pertussis vaccines are highly effective, strongly recommended, and save thousands of lives each year. The efficacy rate depends on the
type of vaccine (acellular or whole cell), the manufacturer, how many other immunizations are given at the same time, and the age of the
patient; various estimates range from preventing disease transmission in 50%95% of people exposed to the disease.
The duration of protection is between five to ten years. This covers childhood, which is the time of greatest exposure and greatest risk of
death from pertussis.For children, the immunizations are commonly given in combination with immunizations against tetanus,
diphtheria, polio and haemophilus influenzae type B, at ages two, four, and six months. A single later booster is given at three to four
years of age.
The pertussis component of the DPT vaccine accounted for most of the minor local and systemic side effects in many vaccinated infants
(such as mild fever or soreness at the injection site).
Unlike some vaccines, immunization against pertussis does not confer life-long immunity. While adolescents and adults rarely die if they
contract pertussis after the effects of their childhood vaccinations have worn off, they may be very sick and may transmit the disease to
infants and other people at much higher risk of death.
To reduce morbidity and spread of the disease, Canada, France, the U.S. and Germany have approved booster shots for adolescents,
adults, or both, but other countries adhere to the tradition of discontinuing pertussis vaccination after the age of seven. Even where they
are available, most adults have not received booster shots and lack immunity to the disease.

Correct Answer. a

(137). CHERNOBYL tragedy is an example of

a. Point source epidemic

b. Propagated epidemic

c. Modern epidemic

d. Continuous epidemic

Solution. Ans-137: (a) Point source epidemic Ref: Read the text below Sol:
There are several outbreak patterns, which can be useful in identifying the transmission method or source, and predicting the future rate
of infection. Each has a distinctive epidemic curve, or histogram of case infections and deaths.
Common source All victims acquire the infection from the same source (e.g. a contaminated water source).
o Continuous source Common source outbreak where the exposure occurs over multiple incubation periods
o Point source Common source outbreak where the exposure occurs in less than one incubation period e.g CHERNOBYL tragedy
Propagated Transmission occurs from person to person.
Outbreaks can also be:
Behavioral risk related (e.g., sexually transmitted diseases, increased risk due t malnutrition)
Zoonotic The infectious agent is endemic to an animal population.
Patterns of occurrence are:
Endemic a communicable disease, such as influenza, measles, mumps, pneumonia, colds, small pox found in a particular region or
population. This means the disease is characteristic of a particular place, or among a particular group, or area of interest or activity
Epidemic when this disease is found to infect "a lot of" people at the same time and may spread through one or several communities.
Pandemic occurs when an epidemic spreads worldwide.

Correct Answer. a

Copyright 2014 Delhi Academy of Medical Sciences, All Rights Reserved. 61/86
(138). In an epidemic first to be done is

a. Identity the cases

b. Confirm the diagnosis

c. Identify the prone people

d. Identify the causative factors.

Solution. Ans-138: (b) Confirm the diagnosis Ref:Park-111 Sol:


Verification of diagnosis is the first step in an epidemic investigation, as it may happen sometime that the report may be spurious, and
arise from misinterpretation of signs and symptoms by the lay public.
It is therefore necessary to have the verification of diagnosis on the spot, as quickly as possible.
It is not necessary to examine all the cases to arrive at a diagnosis.
A clinical examination of a sample of cases may well suffice.

Correct Answer. b

(139). Time interval between invasion of the infection agent and appearance of first sign or symptom is

a. Serial interval

b. Incubation period

c. Quarantine

d. Period of infectivety

Solution. Ans-139: (b) Incubation period Ref:Park-93 Sol:


An infection becomes apparent only after a certain incubation period, which is defined as the time interval between invasion by an
infectious agent and appearance of the first sign or symptom of the disease in question.
During the incubation period, the infectious agent undergoes multiplication in the host.
When a sufficient density of the disease agent is built up in the host, the health equilibrium is disturbed and the disease becomes overt.

Correct Answer. b

(140). The association between disease & risk factor in a case control study is studied by

a. Relative role

b. Attributable role

c. Population attributable role

d. Odds ratio

Solution. Ans-140: (d) Odds ratio Ref:Read the text below Sol:
The odds ratio is a measure of effect size, describing the strength of association or non-independence between two binary data values.
It is used as a descriptive statistic, and plays an important role in logistic regression
Unlike other measures of association for paired binary data such as the relative risk, the odds ratio treats the two variables being
compared symmetrically, and can be estimated using some types of non-random samples.

Correct Answer. d

Copyright 2014 Delhi Academy of Medical Sciences, All Rights Reserved. 62/86
(141). The lack of linolenic acid may causes :

a. Loss of appetite

b. Loss of weight

c. Oedema

d. Raised serum cholesterol

Solution. Ans-141: (d) Raised serum cholesterol Ref.: Park - 415 Sol :
Diets rich in EFA have been reported to reduce serum cholesterol and low-density, lipoproteins

Correct Answer. d

(142). Consider the following statements regarding slow sand filter : (1) Water is coagulated before it is admitted to the filter bed. (2) The
effective diameter of sand particles marking up the top layer in the filter bed is 0.15 to 0.35 mm. (3) 99.9% bacteriological purity is
obtained after filtration. Which of the above statements are correct ?

a. 1, 2 and 3

b. 1 and 2

c. 2 and 3

d. 1 and 3

Solution. Ans-142: (c) 2 and 3 Ref.: Park - 622 Sol :


Steps are involved in the purification of water by rapid sand filters :
o Coagulation the row water is first treated with alum (5-40 mg or more/L).
o Raid mixing
o Flocculation
o Sedimentation
o Filtration

Correct Answer. c

(143). The air-quality standards proposed by the Indian Central Pollution Control Board are based on limits of concentration of :

a. Suspended particulate matter and sulphur dioxide

b. Suspended particulate matter, sulphur dioxide and oxides of nitrogen

c. Suspended particulate matter, sulphur dioxide, oxides of nitrogen and oxidants

d. Suspended particulate matter, sulphur dioxide, oxides of nitrogen and carbon monoxide

Solution. Ans-143: (d) Suspended particulate matter, sulphur dioxide, oxides of nitrogen and carbon monoxide Ref.: Park - 546 Sol :
National Air Quality Monitoring Programme :
Sponsored by central pollution board (CPCB since 1990
Health related pollutant criteria inhalable dust, sulphur dioxide, nitrogen dioxide, lead, hydrogen sulphide, ammonia and PAH.

Correct Answer. d

Copyright 2014 Delhi Academy of Medical Sciences, All Rights Reserved. 63/86
(144). The incubation period of rabies is :

a. Two weeks

b. Two months

c. Two years

d. Variable

Solution. Ans-144: (d) Variable Ref.: Read the text below Sol :
The rabies incubation period is the length of time between infection with the rabies virus and the onset of symptoms.
While the incubation period for rabies may vary from a few days to several years, it usually lasts one to three months.
During the rabies incubation period, a bite from the infected animal does not carry a risk of rabies because the virus is not yet in the
saliva.

Correct Answer. d

(145). Clinical features of lead poisoning include :

a. Wrist drop and delirium

b. Wrist drop and abdominal colic

c. Delirium and insomnia

d. Insomnia and abdominal colic

Solution. Ans-145: (b) Wrist drop and abdominal colic Ref.: Park - 610 Sol :
Clinical feature of lead poisoning or plumbism different in the inorganic and organic lead exposures.

Correct Answer. b

(146). Which one of the following sets of parameters is taken into account in Kuppuswamys socioeconomic status classification ?

a. Housing, education and occupation

b. Housing, occupation and income

c. Education, occupation and income

d. Housing and income

Solution. Ans-146: (c) Education, occupation and income Ref.: Park - 602 Sol :
Socioeconomic status scale :

Correct Answer. c

Copyright 2014 Delhi Academy of Medical Sciences, All Rights Reserved. 64/86
(147). A 12-year-old boy having lost his father a year ago, is caught shoplifting. The boy will be sent to :

a. An orphanage

b. An anganwari

c. A prison

d. A remand home

Solution. Ans-147: (d) A remand home Ref.: Read the text below Sol :
Remand home is an institution to which juvenile offenders between 8 and 14 years may be remanded or committed for detention

Correct Answer. d

(148). All of the following are function of a Primary Health Centre, except

a. Reporting of births and deaths

b. Providing supplementary nutrition to children under 5 years of age

c. Training local Dais and Health Guides

d. Health education

Solution. Ans-148: (b) Providing supplementary nutrition to children under 5 years of age Ref.: Park - 696 Sol :
Functions of primary health centre :
o Medical care
o MCH including family planning
o Safe water supply and basic sanitation
o Prevention and control of locally endemic diseases
o Collection and reporting of vital statistics
o Health education
o National health programmes
o Referral services
o Training of health guides, health workers, local dais and health assistants.
o Basic laboratory services.

Correct Answer. b

(149). The following data relates to immunization performance for a year in a sub-centre having a population of 5000 :
Infants alive at the age of 1 year 100
BCG given 100
DPT1/OPV1 given 80
DPT2/OPV2 given 70
DPT3/OPV3 given 65
Measles given 60
Based on this data, the drop-out rate for fully immunised children will be :

a. 40%

b. 30%

c. 20%

d. 10%

Solution. Ans-149: (a) 40% Ref.: Read the text below Sol :
Drop out rate for fully immunized child = Highest covered antigen dose lowest covered antigen dose/highest covered antigen dose
100 = 100 60/100 100 = 40

Correct Answer. a

Copyright 2014 Delhi Academy of Medical Sciences, All Rights Reserved. 65/86
(150). Annual infection rate in TB is the percentage of :

a. Person converted from tuberculine negative to positive

b. New cases of tuberculosis

c. Sputum positive cases

d. Radiological cases

Solution. Ans-150: (a) Person converted from tuberculine negative to positive Ref.: Park -148 Sol :
Incidence of infection (annual infection rate) :
Also known as tuberculin conversion Index, is one of the best indicators for evaluating the tuberculosis problem and its trends.
It is the percentage of population under study who will be newly infected by Mycobacterium tuberculosis among the non-infected of the
preceding survey during the course of one year.

Correct Answer. a

(151). Under the revised National Tuberculosis Control Programme, a new case is one who has never had treatment for tuberculosis or has
taken antitubercular drugs for less than :

a. 2 weeks

b. 4 weeks

c. 6 weeks

d. 8 weeks

Solution. Ans-151: (b) 4 weeks Ref.: Park -149 Sol :


New case A patient with sputum positive pulmonary tuberculosis who has never had treatment for tuberculosis or has taken
antituberculosis drugs for less than 4 weeks.
Failure case A patient who was initially smear positive, who began treatment and who remained or became smear positive again at 5
months or later during the course of treatment.
Return after default A patient who returns sputum smear positive, after having left treatment for at least 2 months.

Correct Answer. b

(152). According to WHO, all the following diseases require surveillance, except :

a. Chickenpox

b. Yellow fever

c. Malaria

d. Rabies

Solution. Ans-152: (a) Chickenpox Ref.: Read the text below Sol :
WHO notifiable disease yellow fever, cholera, plague.
International surveillance polio, rabies, relapsing fever, salmonellosis, louse-born typhus, influenza, malaria

Correct Answer. a

Copyright 2014 Delhi Academy of Medical Sciences, All Rights Reserved. 66/86
(153). The two important values necessary for describing the frequency distribution of a series of observation are :

a. Mean and standard deviation

b. Mean and range

c. Median and range

d. Median and standard deviation

Solution. Ans-153: (a) Mean and standard deviation Ref.: Park -647 Sol :
Demonstration of a normal distribution when large number of observations of any variable characteristics such as Hb value and make a
frequency distribution with narrow class intervals, we are likely to get a smooth, symmetrical curve.
Such a curve is called normal distribution or Gaussian distribution. The shape of the curve will depend upon the mean and standard
deviation (SD).
Mean + ISD 68.27% or roughly two-thirds of values
Mean + 2SD 95.45%
Mean + 3SD 99.73%

Correct Answer. a

(154). All of the following diagrams can be drawn from quantitative data, except :

a. Frequency curve

b. Scatter diagram

c. Flow diagram

d. Histogram

Solution. Ans-154: (c) Flow diagram Ref.: Mahajan, (6the Ed.), Pg. 20 Sol :
Methods of presentation of data :
o Tabulation (i) Qualitative data, (ii) Quantitative data
Frequency distribution drawings
o Graphs presentation of quantitative continuous or measured data is through graph. Common graphs in use are (1) histogram, (2)
frequency polygon, (3) frequency curve, (4) line chart or graph, (5) cumulative frequency diagram, (6) scatter or dot diagram.
o Diagrams presentation of qualitative discrete or counted data is through diagrams. The common diagrams in use are (1) bar
diagram, (2) Pie or sector diagram, (3) Pictogram or picture diagram, (4) Map diagram or spot map

Correct Answer. c

(155). Concentration of which of the following was same in reduced osmolarity ORS is :

a. Glucose

b. Sodium Chloride

c. Potassium

d. All of above

Solution. Ans-155: (c) Potassium Ref.: Read the text below Sol :
In 2003, WHO/UNICEF changed the ORS formula to a reduced osmolarity version from what it had recommended for more than two
decades.
This change was in response to numerous studies that showed that the standard ORS formula was ineffective in reducing diarrheal stool
output compared to other solutions, including rice water.
Additionally, further studies showed that a reduced osmolarity solution not only decreased stool output, but also resulted in less
vomiting and fewer unscheduled intravenous therapy cases.
Although UNICEF certifies reduced osmolarity ORS for all forms of dehydration, at least one study cautions that for high stool output
cholera-based diarrhea, reduced osmolarity ORS may not sufficiently replenish electrolyte levels, leading to hyponatremia. Though the
actual consequence of this appeared negligible, further study was recommended.
The change reduced the osmolarity of the ORS from 311 mmol/L to 245 mmol/L. The ingredients reduced in concentration were glucose
and sodium chloride. Potassium and citrate concentrations remained the same. The benefits of the reduced osmolarity ORS are reducing
stool volume by about 25 percent, reducing vomiting by nearly 30 percent, and reducing the need for unscheduled intravenous therapy
by 33 percent

Correct Answer. c

Copyright 2014 Delhi Academy of Medical Sciences, All Rights Reserved. 67/86
(156). Nitrates in drinking water indicates :

a. Remote faecal pollution

b. Recent faecal pollution

c. Unpolluted water

d. Water fit for drinking

Solution. Ans-156: (a) Remote faecal pollution Ref.: Read the text below Sol :
Nitrates and nitrites are indicators of remote and recent faecal pollution respectively.
An increase of nitrates in water is often associated with farming fertilizer, pesticide or poor sanitary activities.
The WHO guideline for nitrates in drinking water is established to prevent methaemoglobinaemia (blue babies), which is lethal in
babies.
It can e potentially hazardous with health risks for considerable groups of people and depends on the conversion of nitrates to nitrites
and wells which were shallow, dug of located on large farms, or springs were more likely to have elevated concentrations of nitrates.
Contamination of drinking water by nitrates is an evolving public health concern since nitrates can undergo endogenous reduction to
nitrites (nitrate (III), and nitrosation of nitrites can from N-nitrose compounds, which are potent carcinogens.

Correct Answer. a

(157). National rural health mission is started by prime minister in :

a. 2003

b. 2004

c. 2005

d. 2006

Solution. Ans-157: (c) 2005 Ref.: Read the text below Sol :
The National Rural Health Mission was launched by the Honble Prime Minister on 5th April 2005, to provide accessible, affordable and
accountable quality health services even to the poorest households in the remotest rural regions.
The thrust of the Mission was on establishing a fully functional, community owned, decentralized health delivery system with inter
sectoral convergence at all levels, to ensure simultaneous action on a wide range of determinants of health like water, sanitation,
education, nutrition, social and gender equality.

Correct Answer. c

(158). Which of the following is posted at a subcentre :

a. Voluntary worker

b. Anganwadi workers

c. Trained dai

d. health guide

Solution. Ans-158: (a) Voluntary worker Ref.: Read the text below Sol :
In the public sector, a Sub-health Centre (Sub-centre) is the most peripheral and first contact point between the primary health care
system and the community.
As per the population norms, one Sub-centre is established for every 5000 population in plain areas and for every 3000 population in
hilly/tribal/desert areas.
A sub-centre provides interface with the community at the grass root level, providing all the primary health care services. As sub
centres are the first contact point with the community, the success of any nation wide programme would depend largely on well
functioning sub centres providing services of acceptable standard to the people.
Currently a Sub centre is staffed by one female health worker commonly known as Auxiliary Nurse Midwife (ANM), one Male Health
Worker commonly known as Multi purpose worker (Male) and volunteer worker.
One Health Assistant (Female) commonly known as Lady Health Visitor (LHV) and one Health Assistant (Male) located at the PHC level
are entrusted with the task of supervision of all the Subcentres (generally six subcentres) under a PHC.

Correct Answer. a

Copyright 2014 Delhi Academy of Medical Sciences, All Rights Reserved. 68/86
(159). Number of beds in a CHC is :

a. 20

b. 30

c. 40

d. 50

Solution. Ans-159: (b) 30 Ref.: Parks - 89 Sol :


One PHC is to cover a population of 20000 in Hilly/ Tribal / Difficult areas and 30000 in Plain areas.
As per minimum requirement, a PHC is to be manned by a Medical Officer supported by 14 paramedical and other staff. Under NRHM,
there is a provision for two additional Staff Nurses at PHCs on contract basis. It acts as a referral unit for 6 Sub Centres and has 4 - 6
beds for patients.
The activities of PHC involve curative, preventive, promotive and Family Welfare Services. As per minimum norms, a Community Health
Centre (CHC) is required to be manned by four Medical Specialists i.e. Surgeon, Physician, Gynaecologist and Paediatrician supported by
paramedical and other staff.
It has 30 in-door beds with one OT, X-ray, Labour Room and Laboratory facilities. It serves as a referral centre for 4 PHCs and also
provides facilities for obstetric care and specialist consultations. One CHC is to cover a population of 80000 in Hilly/ Tribal / Difficult
areas and 1,20,000 in Plain areas.

Correct Answer. b

(160). Lepromin test is best for

a. Prognosis

b. Diagnosis

c. Treatment

d. Epidemiological investigations

Solution. Ans 160: (a) Prognosis Ref: Read the text below Sol:
Lepromin test done intradermally, 0.1 ml of lepromin into the inner aspect of forearm of the individual examined on 48 hr and 21 days
Early reaction (Fernandez reaction) inflammation response disappear after 3-4 days, d/t soluble constituent of bacteria. Diameter of
red area is measured
Late reaction (Mitsuda reaction) apparent in 7-10 days
Test read at 21 days, it is d/t type IV hypersensitivity
Diameter of nodule is measured
Value of this test Not a diagnostic test Evaluation of the cell mediated immunity of leprosy patient [not general population] Classification
of the type of disease Prognosis of the disease Now coming to option (D) epidemiological studies Park 271 writes probably the most
important recent advance in the study of the epidemiology of leprosy is the development of sociological tests They have importance in
Detection of subclinical infection Preclinical manifestation Follow up efficacy of drug treatment Includes:
FLA ABS [Fluorescence leprosy Ab absorption test]
Monoclonal ab ab competition test (SALT)
Others Radioimmunoassay, ELISA PGL Ag based [phenolic glycolipid]

Correct Answer. a

Copyright 2014 Delhi Academy of Medical Sciences, All Rights Reserved. 69/86
(161). Plastic wrapper of used syringes is discarded in

a. Red bag

b. Black bag

c. Yellow bag

d. Blue bag

Solution. Ans 161: (b) Black bag Ref: Read the text below Sol:
Black bag is used for collecting general waste i.e., paper, non-infectious plastics, cardboard boxes, and other dry waste generated in hospital
office or in the wards
Colour coding and type of container for disposal of biomedical waste

Correct Answer. b

(162). Shortest incubation period is seen in

a. Hepatitis A

b. HBV

c. Rubella

d. Influenza

Solution. Ans 162: (d) Influenza Ref: Read the text below Sol: Incubation period of various disease are
Hep A 20-30 days [15-45 days]
Hep B < 100 days [45-180 days]
Rubella 18 days [2-3 weeks]
Influenza 18-72 hrs
Varicella/chickenpox 14-16 days [7-21 days]
Measles (Rubeola) 10-14 days
Mumps 18 days [2-3 weeks]
Dipheria 2-6 days
Pertussis 7-14 days [Not more than 3 weeks]
Meningococcal meningitis 3-4 days [2-10 days]
Cholera few hrs 5 days [commonly 1-2 days]
Typhoid fever 10-14 days [3-21 days]

Correct Answer. d

Copyright 2014 Delhi Academy of Medical Sciences, All Rights Reserved. 70/86
(163). About DDT are true except :

a. Contact poison

b. Kills immediately

c. Residual effect persists for 18 months

d. Causes synergistic action with pyrethrin

Solution. Ans 163: (b) Kills immediately Ref: Read the text below Sol: (Dichloro-diphemyl-trichloroethane) 1874-German chemist
Ziedler
Weight amorphous powder, insoluble in water
Para-Para isomer-most active isomer
Primarily A contact poison-act on nervous system
Does not cause immediate death several hours to kill
Residual action up to 18 month
No repellent action
Applied as spray 100-200 mg/sq.foot Regarding Hazard-acc. to WHO there is no valid reason to discontinue the use of DDT for
application in houses against malaria or other disease because Its safety for human use still remains unchallenged despite the
tremendous amount of research. Most space spray contain Pyrethrin & DDT or other synthetic insecticide which are added for
synergistic action

Correct Answer. b

(164). All are larvicidal measures except

a. DDT

b. Gamburia

c. Intermittent irrigation

d. Paris green

Solution. Ans 164: (a) DDT Ref: Read the text below Sol: DDT is NOT recommended for larvicidal operation because long residual effect
and development of resistance. Mosquito control measure integrated approach is now recommended. various methods:-
Anti larval measure
Anti adult measure
Protection against mosquito bites
1. Antilarval (a) Environmental control aiming in reducing breeding places k/s source red and comprises minor engineering method as
filling leveling & drainage of breeding places and water management - Intermittent irrigation - Change in salinity (b) Chemical control -
Mineral oil oil to water like diesel,? kills larvae and pupae - Paris green copper acetoarsenite-emrald green - Stomach poison (must be
ingested by larva for action) - Applied as rotary mixture 2 kg PG + 98 kg diluents - Synthetic insecticide side fenthion, chlorpyrifos,
abate 1 PPM DDT is NOT recommended for larvicidal operation because long residual effect and development of resistance. (c)
Biological control small fish Gambusia affinis & lebister reticulatus also k/s Barbados millions
2. Anti adult:- (a) Residual sprays DDT is insecticide of choice 1-2 gm pure form to 1 sq.mt., 1-3 times/year (b) Space spray form of
mist or fog pyrethrum-Nerve poison skill immediately on mere contact (c) Genetic control-sterile male technique, sex distortion 3.
Protection against mosquito bites (a) Mosquito nets size of pore-diameter = 0.0475 inch & contain 150 holes/sq.inch (b) Screening of
buildings with copper or bronze gauze (c) Repellents Diethyl toluamide, outstanding

Correct Answer. a

Copyright 2014 Delhi Academy of Medical Sciences, All Rights Reserved. 71/86
(165). Test for pasteurization is

a. Phosphatase test

b. Methylene blue reduction test

c. Ethylene blue test

d. None

Solution. Ans 165: (a) Phosphatase test Ref: read the text below Sol:
Phosphotase is an enzyme present in raw milk which should be completely destroyed after adequate pasteurization.
Milk an efficient vehicle for a great variety of disease.
The safety and keeping quality of milk are related to its microbial content
Content is reduced by pasteurization
Definition heating of milk to such temp and for such period of time as are required to destroy any pathogen that may be present while
causing minimal changes in composition. Methods
Holder 63-66C for 30 min cooled quickly to 5C
HTST 72C for 15 sec cooled to 4C for large quantity of milk to be pasteurized
UHT method ultra high temp method heated in 2 stages (in second stage usually under pressure) 125C few seconds only tea
rapidly cooled and bottled.
Advantage: Kills nearly 90% of bacteria including TB, Q fever organism. Not good for thermoduric bacteria and other bacteria spores.
Tests on pasteurized milk Phosphatase test check efficiency of pasteurization raw milk has enzyme phosphatase and is destroyed by
heating

Correct Answer. a

(166). Not a part of SAFE strategy for trachoma

a. Safe excreta disposal

b. Face washing

c. Vit A

d. Azithromycin oral

Solution. Ans 166: c) Vit A Ref: Read the text below. Sol: VISION 2020 The Right to Sight is the common agenda launched by the WHO
and Task Force of International organization to combat blindness, as 80% of global blindness is avoidable. Five conditions that have been
identified as immediate priorities within VISION 2020 are Cataract, trachoma, Onchocerciasis, childhood blindness and refractive errors
and low vision. Mnemonic: Vision 2020 includes COReCT
Cataract
Onchocerciasis
Refractive error
Childhood blindness
Trachoma It is expected that through SAFE strategy it will be possible to eliminate trachoma by year 2020. It includes Surgery first
component of the strategy for correction of trichiasis and entropion. Antibiotics Zithromax (azithromycin) single dose oral medication is
used (Zithromax is donated by Pfizer Inc.) Tetracycline eye ointment was use before Pfizer donated Zithromax. Face washing breaks
the cycle of reinforcement and prevents transmission of disease. Environmental Improvement - Poverty and poor living conditions
contribute to high rates of blinding trachoma. Environmental improvement includes Availability of improved water supplies. Improved
household sanitation, particularly safe disposal of feces.

Correct Answer. c

Copyright 2014 Delhi Academy of Medical Sciences, All Rights Reserved. 72/86
(167). Not a goal for national population policy 2015

a. TFR to replacement levels by 2015

b. IMR 30/1000

c. MMR 100/100000

d. 100% registration of birth, death and marriage

Solution. Ans 167: a) TFR to replacement levels by 2015 Ref: Read the text below. Sol:
There are no goals in NPP 2000 for the year 2015. The TFR goal has no time limit set in this policy. It is to be achieved as soon as
possible.
National Health Policy-2002 by Ministry of Health & Family Welfare
Objectives
Acceptable std of good health in general population
Decentralized public health system by New infrastructure in the existing institutions
Equitable access to health service
Primary to preventive and 1st line curative initiative at primary health level
Focus on do with contribute to burden as TB, malaria, blindness and HIV/AIDS

Correct Answer. a

(168). Pearl index is for

a. Contraception efficacy

b. Contraceptive failure

c. Contraceptive quality.

d. None

Solution. Ans 168: a) Contraception efficacy Ref: Read the text below. Sol: Contraceptive efficacy in general assessed by measuring the
number of unplanned pregnancies that occur during a specified period of exposure and use of a contraceptive method Methods are :
Pearl index(PI) Life table analysis PI = No. of failures per 100 women year of exposure

Denominator = for full term pregnancy = 10 month For abortion = 4 month PI = based on specific exposure (1 year) and fails to
accurately compare methods at various duration of exposure
Life table analysis Overcomes the above said limitation and calculates a failure rate for each month

Correct Answer. a

(169). True statement regarding NPU is

a. Nitrogen retained in body devided by nitrogen intake multiplied by 100

b. Best indicator of protein quality.

c. Does not depend on energy intake.

d. None

Solution. Ans 169: a) Nitrogen retained in body devided by nitrogen intake multiplied by 100 Ref: Read the text below. Sol:
NPU is nitrogen retained in body devided by nitrogen intake multiplied by 100
The Protein Digestibility Corrected Amino Acid Score is a more modern rating for determining protein quality.

Correct Answer. a

Copyright 2014 Delhi Academy of Medical Sciences, All Rights Reserved. 73/86
(170). True about maximum isolation period of enteric fever is?

a. Up to 3 days after starting the treatment

b. Up to 5 days after starting the treatment

c. Till 3 consecutive negative stool and urine samples are obtained

d. Till 5 consecutive negative stool and urine samples are obtained

Solution. Ans 170: c) Till 3 consecutive negative stool and urine samples are obtained Ref: Read the text below Sol: Typhoid fever and
paratyphoid fever are called as Enteric fever Major cause S. typhi Reservoir = man only Carrier = temporary, or chronic Av. Carrier
rate is around = 3 percent Chronic cancer = who excrete bacilli for more than a year after a clinical attack it is due to persistence of
organism in Gall bladder and billiary tract Age = Any age (max. 3-19 yrs) Sex = Males > females, but carrier female> male Immunity by
Ab against Ag O (Somatic Ag) & Flagellar Ag (H) IP = 10-14 days [3 days 3 weeks) Clinical features = Abdominal pain, constipation, in
proderm then 7-10 days after exhaustion and Pea Soup diarrhea Diagnosis = Isolation from blood, BM, stool, blood culture mainstay
Serology felix-widal test New diagnostic test IDL tubex = IgMO 9 ab from patient within few min Typhidot 3 hrs time IgM, IgG
Typhidot M IgM only Dipstick test - IgM Isolation As a rule cases should be isolated till three bacteriologically negative stool and
urine reports are obtained on three separate days Treatment fluoquinolones Disinfection of stool 5% cresol for 2 hr

Correct Answer. c

(171). For one case of measles, the subclinical case/s expected in the community is/are :

a. Zero

b. 10

c. 100

d. 1000

Solution. Ans-171: (a) Zero Ref.: Read the text below Sol :
This has been asked innumerable times. However this time they have changed the language a bit.
What they want to ask is about subclinical cases or carriers.
The only source of infection is a case of measles. Carriers are not known to occur.

Correct Answer. a

(172). The approximate calorific value of 2 slices of bread, in kilocalories is :

a. 70

b. 170

c. 270

d. 370

Solution. Ans-172: (b) 170 Ref.: Read the text below Sol :
100 gms of white bread contains 245 calories, and one slice contains around 25 gms.
So two slices of white bread = 50 gms = 245/2 Calories = 122.5 Calories (1 Calorie = 1 kilo calorie, note the difference between capital
C and small c)

Correct Answer. b

Copyright 2014 Delhi Academy of Medical Sciences, All Rights Reserved. 74/86
(173). Father of (modern) epidemiology is :

a. Fracastorius

b. Edwin Chadwick

c. John Snow

d. James Lind

Solution. Ans-173: (c) John Snow Ref.: Read the text below Sol :
Epidemiology is the study (or the science of the study) of the patterns, causes, and effects of health and disease conditions in defined
populations. It is the cornerstone of public health, and informs policy decisions and evidence-based medicine by identifying risk factors
for disease and targets for preventive medicine. Epidemiologists help with study design, collection and statistical analysis of data, and
interpretation and dissemination of results (including peer review and occasional systematic review).
Dr. John Snow is famous for his investigations into the causes of the 19th century cholera epidemics, and is also known as the father of
(modern) epidemiology

Correct Answer. c

(174). All affects equilibrium in hardy- weinbergs law except-

a. Random mating

b. Gene mutation

c. Gene flow migration

d. Small population

Solution. Ans-174: (a) Random mating Ref: Read the text below Sol : The Hardy-Weinberg Law of Genetic Equilibrium In 1908 G. Hardy
and W. Weinberg independently proposed that the frequency of alleles and genotypes in a population will remain constant from
generation to generation if the population is stable and in genetic equilibrium. Five conditions are required in order for a population to
remain at Hardy-Weinberg equilibrium: 1. A large breeding population 2. Random mating 3. No change in allelic frequency due to
mutation 4. No immigration or emigration 5. No natural selection The Hardy-Weinberg Equation To estimate the frequency of alleles in a
population, Hardy-Weinberg equation, states that: If, p = the frequency of the dominant allele (represented here by A) q = the frequency
of the recessive allele (represented here by a) For a population in genetic equilibrium: p + q = 1.0 (The sum of the frequencies of both
alleles is 100%.)
(p + q) 2 = 1. So, p2 + 2pq + q2 = 1 The three terms of this binomial expansion indicate the frequencies of the three genotypes: p2 =
frequency of AA (homozygous dominant) 2pq = frequency of Aa (heterozygous) q2 = frequency of aa (homozygous recessive)

Correct Answer. a

(175). All of the following are true regarding acute attack of Polio, except

a. Fecal sample is carried in cold box

b. Three consecutive stool samples are positive

c. Investigation done in 48 hours

d. Throat swab is taken for diagnosis.

Solution. Ans-175: (b) Three consecutive stool samples are positive Ref: Read the text below Sol :
The process of sending stool sample of a suspected case of polio to lab is known as Reverse cold chain.
Two samples should be taken preferably with in two weeks of onset of paralysis. Amount should be about 8 gms or distal phalange of
thumb.

Correct Answer. b

Copyright 2014 Delhi Academy of Medical Sciences, All Rights Reserved. 75/86
(176). If prevalence of diabetes is 10%, the probability that three people selected at random from the population will have diabetes is :

a. 0.01

b. 0.03

c. 0.001

d. 0.003

Solution. Ans-176: (a) 0.01 Ref: Read the text below Sol :
There are two rules of probability, the addition law and the multiplication law.
Probability of one person having diabetes is p = 1/10 = 0.1.
The probability of all 3 having diabetes can be calculated using the multiplication law or probability. It will be pxpxp = 0.1 x 0.1 x 0.1 =
0.001

Correct Answer. a

(177). True about BCG vaccination is

a. Danish 1331 strain is used for vaccine production

b. Both distill and normal saline can be used as diluents

c. Site should be cleaned with spirit

d. None

Solution. Ans 177: a) Danish 1331 strain is used for vaccine production Ref: Read the text below Sol: WHO recommended Danish 1331
strain,
Vaccine must be protected from exposure to light during storage and in fields (red/black double layered cloth)
Only normal saline is recommended not distill water (irritant) for reconstitution
Used within 3 hr
mg in 0.1 ml vol. with tuberculin syringe [Omega microstat syringe fitted with 1 cm steel 26 gauze intradermal needle] just above
insertion of the deltoid muscle
Vaccine should not be contaminated with antiseptic or detergent
If alcohol is used to swab the skin-allowed to evaporate before the vaccine is given

Correct Answer. a

(178). Cyclodevelopmental propagation is seen in:

a. Filaria

b. Malaria

c. Plague

d. Yellow fever

Solution. Ans 178: a) Filaria Ref: Read the text below. Sol: Transmission of arthropod-borne disease Three types of transmission cycles
are involved in the spread of arthropod born disease
Direct contact-arthropod are transferred directly from man to man e.g. scabies, pediculosis
Mechanical mech. By arthropod -diarrhoea, dysentery, lymphoid, food poisoning and trachoma by the housefly
Biological-when disease agent multiply or dev change or both in arthropod host
It is of 3 types Propogative only multiplication no development e.g. plaque bacilli cyclopropogative both dev and multiplication e.g.
malaria Cyclo developmental only development occur e.g. filarial, guinea worm

Correct Answer. a

Copyright 2014 Delhi Academy of Medical Sciences, All Rights Reserved. 76/86
(179). RCT include all except

a. Provide comparable base line chart.

b. Drop outs are excluded from study.

c. Sample size depends on study.

d. Assure comparability.

Solution. Ans 179: b) Drop outs are excluded from study. Ref: Read the text below. Sol: The drop-outs may also be included in the final
analysis of RCTs and they are not necessarily excluded from it. RCT the characteristic feature in nutshell
Study conducted under a strict protocol which prevents bias and reduce the source of error
Target population population to which the finding of trial are applicable
Study population population derived from target or reference population. The actual population taking part in studies
Randomization statistical procedure by which population are distributed to study and control group use attempt to eliminate bias
Assure comparability
Thus randomization eliminates selection bias

Correct Answer. b

(180). Breast milk, false is

a. Max secretion at 12 months

b. Is used as reference protein

c. Breast feeding should be initiated within an hour of birth

d. Vit D is present in Breast milk in soluble form

Solution. Ans 180: a) Max secretion at 12 months Ref: Read the text below Sol: P

Breast feeding should be initiated within an hour of birth Br milk has 1.1 gm protein/100 ml,70 kcal/100 ml
Advantage
Safe, clean, hygenic, cheap, correct temperature.
Antimicrobial factors, IgA, ant streptococcal factor lysozyme, lactoferin
Easily digested
Promote bonding between mother & infants
Note Vit D is present in Breast milk in soluble form

Correct Answer. a

Copyright 2014 Delhi Academy of Medical Sciences, All Rights Reserved. 77/86
(181). True about maximum isolation period of enteric fever is?

a. Up to 3 days after starting the treatment

b. Up to 5 days after starting the treatment

c. Till 3 consecutive negative stool and urine samples are obtained

d. Till 5 consecutive negative stool and urine samples are obtained

Solution. Ans 181: c) Till 3 consecutive negative stool and urine samples are obtained Ref: Read the text below Sol: Typhoid fever and
paratyphoid fever are called as Enteric fever Major cause S. typhi Reservoir = man only Carrier = temporary, or chronic Av. Carrier
rate is around = 3 percent Chronic cancer = who excrete bacilli for more than a year after a clinical attack it is due to persistence of
organism in Gall bladder and billiary tract Age = Any age (max. 3-19 yrs) Sex = Males > females, but carrier female> male Immunity by
Ab against Ag O (Somatic Ag) & Flagellar Ag (H) IP = 10-14 days [3 days 3 weeks) Clinical features = Abdominal pain, constipation, in
proderm then 7-10 days after exhaustion and Pea Soup diarrhea Diagnosis = Isolation from blood, BM, stool, blood culture mainstay
Serology felix-widal test New diagnostic test IDL tubex = IgMO 9 ab from patient within few min Typhidot 3 hrs time IgM, IgG
Typhidot M IgM only Dipstick test - IgM Isolation As a rule cases should be isolated till three bacteriologically negative stool and
urine reports are obtained on three separate days Treatment fluoquinolones Disinfection of stool 5% cresol for 2 hr

Correct Answer. c

Copyright 2014 Delhi Academy of Medical Sciences, All Rights Reserved. 78/86
(182). According to WHO clinical staging system for HIV infection and redisease in adults ;Stage 4 includes ?

a. Persistent generalized lymphdenopathy

b. Performance scale 1: asymptomatic, normal activity

c. Pulmonary TB

d. Pneumocystis carinii pneumonia

Solution. Ans 182: (d) Pneumocystis carinii pneumonia Ref:Read the text below Sol:

Correct Answer. d

Copyright 2014 Delhi Academy of Medical Sciences, All Rights Reserved. 79/86
(183 Categorisation of countries is done on the basis of implementation of
). the dots strategy and case notification rate. Pilot phase includes :

a. Category1

b. Category 2

c. Category 3

d. Category4

Solution. Ans 183: (4) Category 4 Ref:Read the text below Sol

Correct Answer. d

(184). Which one of the following statements regarding the chi- squared test is true?

a. It is used as an alternative to the t-test to determine difference between two means

b. It can be used to test the difference between two nominal variables

c. The smaller the value of the chi- squared value, the more likely it is to be significant

d. The number of degrees of freedom of the test for a two by two table is 2.

Solution. Ans 184: (b) It can be used to test the difference between two nominal variables Ref:Read the text below Sol
The chi- squared test is used to determine the difference between observed and expected frequencies or between two or more
frequencies.
The calcutaed chi square value is compared with critical value of 2 from tables, at a predetermined significance level and appropriate
degrees of freedom.
The larger the 2 value, the smaller the probability P. and the more likely it is that the null hypothesis is untrue.
The statistical significance of the results also depends on the size of the table, represented by the degrees of freedom.
The number of degrees of freedom for a two by two table is 1.

Correct Answer. b

(185). Early diagnosis and treatment is included in : -

a. Primary prevention

b. Secondary prevention

c. Tertiary prevention

d. None

Solution. Ans 185: (b) Secondary prevention Ref:Read the text below Sol: CONCEPTS OF PREVENTION The three phases of prevention
are: (i) primary prevention (ii) secondary prevention (iii) tertiary prevention. These phases are further categorised into five levels of
prevention as follows: Primary prevention Health promotion Specific protection Secondary prevention Early diagnosis and treatment
Tertiary prevention. Disability limitation Rehabilitation

Correct Answer. b

Copyright 2014 Delhi Academy of Medical Sciences, All Rights Reserved. 80/86
(186). Which one of the following statistical tests requires a normal population distribution?

a. Wilcoxon rank sum test

b. Spearmen's rank correlation

c. Chi-squared test

d. Variance estimation

Solution. Ans 186: (d) Variance estimation Reference Read the text below. Sol:
Non parametric tests make no assumptions about the underlying distribution of the sample.
Parametric tests assume that the population is normally distributed, and that the variance of the samples is the same.
Spearman's ad kendall's rank correlation coefficients are the non- parametric alternatives to pearson's correlation coefficient.

Correct Answer. d

(187). The set goal for malaria control in the country during the Tenth Five Year plan is: : -

a. API 11 or less

b. ABER over 10 percent

c. 50 percent reduction in morbidity due to malaria by 2007

d. 75 percent reduction in morbidity due to malaria by 2010

Solution. Ans 187: (2) ABER over 10 percent Reference P19-349 Sol: The set goals for malaria control in the country during the Tenth
Five Year Plan are: a. ABER over 10 per cent b. API 1.3 or less c. 25 percent reduction in morbidity and mortality due to malaria by 2007
and 50 percent by 2010.

Correct Answer. b

(188). Sling psychrometer is used for assessing:

a. Air velocity

b. Radiant heat

c. Relative humidity

d. Air temperature

Solution. Ans 188: (c) Relative humidity Ref:Read the text below Sol: SLING PSYCHROMETER
Relative humidity can be measured by an instrument called a hygrometer.
The simplest hygrometer - a sling psychrometer - consists of two thermometers mounted together with a handle attached on a chain.

Correct Answer. c

Copyright 2014 Delhi Academy of Medical Sciences, All Rights Reserved. 81/86
(189). Which one of the following statements about a type 1 error is true?

a. It is the acceptance of null hypothesis that is actually true

b. It is also called the error

c. It is used to help determine an appropriate sample size for a study

d. It is equal to 1 mines the error

Solution. Ans 189: (c) It is used to help determine an appropriate sample size for a study Ref Read the text below Sol:
Type I error is used to help determine the sample size of a study and to test the null hypothesis, which is often rejected at the arbitrary
cut off of 5% probability due to chance.
The error is the acceptance as true of false null hypothesis, and the formula' 1 minus the error, is used to calculate the power of the
study.

Correct Answer. c

(190). Choose incorrect statement regarding case control studies : -

a. Relatively easy to carry out

b. Rapid and inexpensive

c. Risk factors can be identified

d. Allows the study of single aetiological factor

Solution. Ans 190: (d) Allows the study of single aetiological factor Ref Read the text below Sol: CASE CONTROL STUDIES Advantages
and disadvantages of case control studies Advantages 1. Relatively easy to carry out 2. Rapid and inexpensive (compared with cohort
studies) 3. Require comparatively few subjects 4. Particularly suitable to investigate rare diseases or diseases about which little is
known. But a disease which is rare in the general population (e.g., leukaemia in adolescents) may not be rare in special exposure group
(e.g. prenatal X-rays). 5.No risk to subjects 6.Allows the study of several different aetiological factors (e.g., smoking, physical activity and
personality characteristics in myocardial infarction) 7.Risk factors can be identified. Rational prevention and control programmes can be
established

Correct Answer. d

(191). Most common cause of maternal mortality in India?

a. Anemia in pregnancy

b. Haemorrhage

c. Heart disease

d. Pregnancy induced hypertension

Solution. Ans 191: (b) Haemorrhage Ref:Read the text below Sol: Maternal mortality rate measures the risk of women dying from
puerperal cause.

Correct Answer. b

Copyright 2014 Delhi Academy of Medical Sciences, All Rights Reserved. 82/86
(192). Which one of the following statements about retrospective and prospective studies is true?

a. In a prospective study the cohort originally selected consists of people who are found to have the disease

b. Prospective studies allow direct determination of incidence rates

c. A retrospective study involves a survey of the prevalence of the disease in different strata of the population

d. The retrospective approach has the advantage that there is little or no bias in the assessment of exposure to the suspected factor

Solution. Ans 192: (b) Prospective studies allow direct determination of incidence rates RefRead the text below. Sol:
In a prospective cohort study, exposed and non exposed patients are identified and followed over time to determine the incidence of a
specific clinical disease or event.
For example, a population of smokers and non smokers are followed up to provide comparison rates for lung cancer or heavy disease.
Cross sectional studies provide information on disease prevalence in a population. case- control studies compare individuals with and
without a disease to determine possible associations or risk factors for the disease in question.
Bias may influence the recall of exposure in these studies if possible associations are known, such as the association between cigarette
smoking and lung cancer.
A case control study is relatively easy and inexpensive to conduct because long term follow up is not required, and this type of study is
therefore suitable for studying rare disease.

Correct Answer. b

(193). WHO definition of Low vision is?

a. Vision more than 6/60 and less than 3/60 in best eye

b. Vision more than 6/60 and less than 6/18 in better eye

c. Vision more than 3/60 and less than 6/18 in better eye

d. Vision more than 3/60 and less than 3/18 in best eye

Solution. Ans 193: (b) Vision more than 6/60 and less than 6/18 in better eye Ref:Read the text below Sol:

Correct Answer. b

Copyright 2014 Delhi Academy of Medical Sciences, All Rights Reserved. 83/86
(194). Which one of the following statements is true?

a. The standard error of the mean (SEM) is generally larger than the standard deviation (SD).

b. The standard error of the mean provides a measure of the spread of observations around the mean

c. The mean and standard deviation of random sample will generally be different from the mean and standard deviation of the
population.

d. The coefficient of variation is derived from the range.

Solution. Ans 194: (c) The mean and standard deviation of random sample will generally be different from the mean and standard
deviation of the population. RefRead the text below. Sol:
The standard error (SE) measures the variability of sample statistic in relation to the true population characteristic( how accurate the
sample mean is as an estimate of the population mean.).
The SEM is generally smaller than the standard deviation (SD).
The standard deviation is the measure of the variability of the observations.

Correct Answer. c

(195). Delphi method is used for

a. Couple Protection rate

b. Qualitative analysis of data

c. Quantitative analysis of data

d. None.

Solution. Ans 195: (b) Qualitative analysis of data Reference Read the text below Sol: Delphi method
The Delphi method is a systematic interactive forecasting method for obtaining forecasts from a panel of independent experts

Correct Answer. b

(196). The set goal for malaria control in the country during the Tenth Five Year plan is: : -

a. API 1.3 or less

b. ABER over 5 percent

c. 50 percent reduction in morbidity due to malaria by 2007

d. 75 percent reduction in morbidity due to malaria by 2010

Solution. Ans196: (a) API 1.3 or less Ref:Read the text below Sol: The set goals for malaria control in the country during the Tenth Five
Year Plan are: a. ABER over 10 per cent b. API 1.3 or less c. 25 percent reduction in morbidity and mortality due to malaria by 2007 and
50 percent by 2010.

Correct Answer. a

Copyright 2014 Delhi Academy of Medical Sciences, All Rights Reserved. 84/86
(197). Both sexes of which insect bite and transmit the disease : -

a. Anopheles

b. Stegomyia

c. Mansonia

d. Rat fleas

Solution. Ans197: (d) Rat fleas Reference P19-245 Sol:


The commonest and the most efficient vector of plague is rat flea, X. cheopis, but other fleas may also transmit the infection, e.g., X.
astia, X. brasiliensis and Pulex irritans (human flea}.
Both sexes of the flea bite and transmit the disease.
Anopheles,Aedes(stegomyia) and Mansonia all belongs to family mosquito,in which females are haematophagus while males never bite,
they subsist on plant juices.

Correct Answer. d

(198). Choose incorrect statement regarding case control studies : -

a. Minimum ethical problem

b. No risk to subjects

c. Validation of information obtained is easy

d. Not suited to the evaluation of therapy

Solution. Ans198: (c) Validation of information obtained is easy Ref: Read the text below Sol: CASE CONTROL STUDIES Advantages and
disadvantages of case control studies Advantages 1. Relatively easy to carry out 2. Rapid and inexpensive (compared with cohort studies)
3. Require comparatively few subjects 4. Particularly suitable to investigate rare diseases or diseases about which little is known. But a
disease which is rare in the general population (e.g., leukaemia in adolescents) may not be rare in special exposure group (e.g. prenatal
X-rays). 5.No risk to subjects 6.Allows the study of several different aetiological factors (e.g., smoking, physical activity and personality
characteristics in myocardial infarction) 7.Risk factors can be identified. Rational prevention and control programmes can be established
8.No attrition problems, because case control studies do not require follow-up of individuals into the future 9. Ethical problems minimal

Correct Answer. c

(199). PERT (Programme evaluation & review technique) is related with:

a. Cost-benefit analysis

b. Cost-effective analysis

c. Network analysis

d. Systems analysis

Solution. Ans 199 : (c) Network analysis Ref Read the text below Sol: Quantitative methods are derived from fields of economics,
operational research and budgeting. 1. Cost benefit analysis 2. Cost-effective analysis 3. Cost accounting 4. Input-output analysis 5.
Model 6. Systems analysis

PERT (Programme evaluation & review technique) CPM (critical path method) 8. Planning programming - budgeting system (PPBS) 9.
Work sampling 10. Decision making

Correct Answer. c

Copyright 2014 Delhi Academy of Medical Sciences, All Rights Reserved. 85/86
(200). A term applied to several types of facilities for rearing children other than in their natural Families is :

a. Orphanages

b. Borstals

c. Remand homes

d. Foster homes

Solution. Ans-200: (d) Foster homes Reference P-472 Sol: FOSTER HOMES
Foster care is a term applied to several types of facilities for rearing children other than in their natural Families.
The good foster home will provide the child with security,love and affection that is needed.

Correct Answer. d

Test Answer
1.(c) 2.(b) 3.(c) 4.(b) 5.(b) 6.(d) 7.(b) 8.(a) 9.(c) 10.(a)

11.(b) 12.(c) 13.(d) 14.(c) 15.(a) 16.(b) 17.(a) 18.(c) 19.(c) 20.(c)

21.(a) 22.(c) 23.(c) 24.(c) 25.(a) 26.(a) 27.(c) 28.(a) 29.(d) 30.(a)

31.(a) 32.(c) 33.(b) 34.(b) 35.(a) 36.(d) 37.(a) 38.(b) 39.(b) 40.(b)

41.(b) 42.(a) 43.(d) 44.(d) 45.(c) 46.(d) 47.(c) 48.(a) 49.(c) 50.(c)

51.(a) 52.(d) 53.(d) 54.(b) 55.(d) 56.(d) 57.(a) 58.(a) 59.(d) 60.(c)

61.(c) 62.(b) 63.(a) 64.(a) 65.(b) 66.(a) 67.(a) 68.(d) 69.(b) 70.(b)

71.(a) 72.(c) 73.(c) 74.(b) 75.(c) 76.(c) 77.(b) 78.(c) 79.(b) 80.(b)

81.(d) 82.(d) 83.(c) 84.(d) 85.(d) 86.(c) 87.(b) 88.(b) 89.(a) 90.(d)

91.(a) 92.(d) 93.(c) 94.(a) 95.(b) 96.(c) 97.(a) 98.(b) 99.(b) 100.(c)

101.(d) 102.(c) 103.(d) 104.(a) 105.(c) 106.(a) 107.(b) 108.(a) 109.(c) 110.(d)

111.(c) 112.(a) 113.(c) 114.(d) 115.(c) 116.(c) 117.(a) 118.(d) 119.(a) 120.(c)

121.(c) 122.(c) 123.(a) 124.(a) 125.(b) 126.(c) 127.(a) 128.(d) 129.(a) 130.(a)

131.(b) 132.(b) 133.(b) 134.(a) 135.(d) 136.(a) 137.(a) 138.(b) 139.(b) 140.(d)

141.(d) 142.(c) 143.(d) 144.(d) 145.(b) 146.(c) 147.(d) 148.(b) 149.(a) 150.(a)

151.(b) 152.(a) 153.(a) 154.(c) 155.(c) 156.(a) 157.(c) 158.(a) 159.(b) 160.(a)

161.(b) 162.(d) 163.(b) 164.(a) 165.(a) 166.(c) 167.(a) 168.(a) 169.(a) 170.(c)

171.(a) 172.(b) 173.(c) 174.(a) 175.(b) 176.(a) 177.(a) 178.(a) 179.(b) 180.(a)

181.(c) 182.(d) 183.(d) 184.(b) 185.(b) 186.(d) 187.(b) 188.(c) 189.(c) 190.(d)

191.(b) 192.(b) 193.(b) 194.(c) 195.(b) 196.(a) 197.(d) 198.(c) 199.(c) 200.(d)

Copyright 2014 Delhi Academy of Medical Sciences, All Rights Reserved. 86/86

Das könnte Ihnen auch gefallen